Enter GreedCare, where the White Coat is Supplanted by the Grey Pinstripe of Corporate Conglomerates

The first in my series of articles about how healthcare has become hellcare.

By Mike Gorback, MD, for WOLF STREET:

It’s been a long time since I wrote an article for Wolf Street. Wolf and I go back to the Testosterone Pit days and some of the old timers here might recall my pieces. In 2016 I ran into a series of personal setbacks and stopped participating here. Wolf pinged me about it. A few months ago, we finally met in person over dinner in SF to “consummate” our long-term virtual friendship. I agreed to try to crank up the engine. The initial plan was too ambitious. I wanted to spill everything I know about how healthcare had become hellcare.

That was ambitious and unrealistic. The landscape changes so fast and the system is so unstable that it’s like trying to drink from a fire hose during an earthquake.  After several discussions with Wolf, I decided to adopt the philosophy of “If you’re going to eat an elephant take small bites.” Wolf suggested that I draw upon my personal experiences. He’s a smart mofo, Wolf.

My approach is going to be a series of vignettes based on my personal experiences seen through the lens of over 4 decades in medicine. I hope you enjoy and learn from reading about my adventures and misadventures.  

Here we go.

I had the same family doctor for 20 years. I picked him because as a member of the medical community I not only had firsthand knowledge of his reputation for quality of care but I had seen his patients and could evaluate his style.

Most of you can’t do that. You’re at the mercy of sites like Healthgrades or Vitals or Google, where patients can leave nasty reviews and the doctor is often defenseless due to patient privacy laws.

Or you’re left with referrals and/or layperson recommendations. There’s no substitute for personal observation by a well-informed observer. I’m the latter; most of you are the former.

Now we cut to the chase with a very simple example from my personal experience. To me it’s simple because I’ve lived in this environment for many years. Just imagine what complicated looks like. Wait – don’t. You’ll sleep better

Things started to run into the ditch when my doctor adopted electronic medical records. Although this is a complicated topic for another day, EMR is not something the medical community clamored for. It was shoved down our throats by the ObamaCare legislation. It imposed large costs and decreased productivity on physicians and hospitals.

I refused to cave in and as a result I got the stick instead of the carrot. Not a bad decision since the carrot turned out to be rotten and the stick (2% cut in Medicare fees) was nowhere near the cost of a loss of 20% in productivity with EMR.

Looking back, do you recall a drop-off in quality time with your doctor after 2010? EMR.

I put up with the diminished level of service for a few years. Then my doctor merged with a corporate conglomerate.

Enter GreedCare, where the white coat is supplanted by the grey pinstripe.

It was during the pandemic. I needed a refill of my BP meds. I did a telemedicine visit. The doctor couldn’t check my BP. He couldn’t do a damn thing except talk.

My bill was $255. Medicare paid $114 and since I hadn’t met my Medicare deductible, they wanted me to pay the balance.

Would you know how this happened and how to fight it or would you have cut a check for the balance?

Being in the business I knew that $114 was what Medicare paid for a level 3 office visit. This was not an office visit. It was telemedicine. I did telemedicine during the lockdown and Medicare paid me about $44 as a specialist. For primary care (my family doctor) it’s a few dollars less.

What happened? Besides the lack of basic information such as ICD code (diagnosis) and CPT code (type of service rendered) the bill did not include POS (place of service).

By now it should be apparent that I was billed for a visit in the office and not a telemedicine visit. $114 is the Medicare fee for a level 3 office visit, which entails a physical examination. Not by telemedicine.

I’ll pose the rhetorical question once again: would you have figured this out?

I emailed my doctor who said since the merger he wasn’t involved in the billing. I billed in-house (i.e., we did our own billing). If you had a billing complaint, I handed it to my office manager and she took care of it personally. I ignored the bills from my family doctor.

Finally, the day came when I was turned over to a collection agency. I wrote to the collection agency with my analysis of the billing. Was this greed or ignorance? I suspected the former.

My reply was simple. I explained the “error” and pointed out that they had already been paid almost triple the true fee. I offered them a choice: go away or duke it out, which would entail a formal complaint to the Office of the Inspector General at Medicare. Never heard from them again.

How many people are ripped off by this kind of exploitation? My guess is a shit ton. What can you do about it? I have no idea. I’ve thought about offering a service to evaluate bills but at this level even a 40% commission on savings isn’t worth the time and effort. There are companies that do this for large amounts such as hospital bills but I have no experience with them.

Be sure to request your EOB (the Explanation of Benefits). This will lay out the CPT and ICD codes, the POS, the bill, the contractual write-off, what insurance paid, and what you owe. The EOB is provided by the insurer. You can try asking the billing company as well.

Asking God to strike the insurance company dead will be a subject for another post.  Back in the day, in 2014, I wrote one about how buying insurance was like a pig in a poke.

What happened with my family doctor? He’s still capable of being a good doctor but IMHO he’s damaged goods. I have a new doctor now.

Stay tuned for my Adventures in Cardiology, how online reviews are manipulated, and why I filed a HIPAA complaint against CVS/CareMark/Aetna.

I’ve often said that you can tell how your life is going by how many lawyers you have. I’m down to one and soon to be zero. I hope to channel that freedom of energy into Wolf Street.

It’s not one blade of grass that stops the croquet ball. It’s a bunch of blades of grass. Let’s be that lawn that stops the croquet ball.

Mike Gorback (screw the formal MD crap. We’re all in this together).

Enjoy reading WOLF STREET and want to support it? You can donate. I appreciate it immensely. Click on the beer and iced-tea mug to find out how:

Would you like to be notified via email when WOLF STREET publishes a new article? Sign up here.

  228 comments for “Enter GreedCare, where the White Coat is Supplanted by the Grey Pinstripe of Corporate Conglomerates

  1. GSH says:

    Welcome back! Very informative. Shedding some light onto the medical industrial complex is sorely needed.

    • Lisa says:

      I am a behavioral health provider. For me, whether I’m office or Telehealth I am providing the same exact service as I do not need to take BP, or do physical tests. Insurance companies tried to pay us less or not allow telehealth altogether. We fought hard to get it to pass here in IL so we could get paid the same rate as an office visit. It is a different billing situation with medical doctors as it’s not an actual exam but a consultation. You must scrutinize your medical bills, always. I find errors all the time. Northwestern Health was charging a fee that was put in place for Covid. Basically it’s a code that is supposed to cover Covid supplies. But insurance won’t pay all of it. They billed 50.00 because the dr office claimed they used 50.00 worth of supplies for one office visit. They didn’t. I brought my own mask and nothing was any more clean pre Covid. Was it for checking my temp? It’s a scam! I told the billing dept I would. It pay it and for them to justify the amount they were charging.
      Since they couldn’t they told me they would remove the fee. I also threatened to file a fraud complaint with my insurance company. You can file for fraud if you see something is off. I was also charged once for an office visit when it was a well visit. The dr asked me if I needed a certain rx that I stopped taking years ago. I said no. Then she made me fill out a depression inventory which I didn’t need. Then I got billed. It was supposed to be a well visit and free. I asked why and was told because we talked abt my depression. I said that is not true. I did not bring up the concern and therefore didn’t fall under the well visit category since I didn’t bring it up. They removed it. Please also be aware of what is and is not covered under your well visit. You could easily be charged (scammed). As a provider I understand these charges. The consumer also must act as if they are making a purchase at a store. Do you buy something without knowing how much it costs? You must do this when you go to the doctor or emergency room (if conscious).
      It sucks (the life out of you).

      • Rg says:

        Yes. The wellness visit is pretty much a scam. Just a way to bill Medicare.

        My elderly mother was duped into going once and would never go again.

        At her age it was a physical challenge to make it to the appointment anyway.

    • cas127 says:

      Wolf/Mike,

      Yes!…doing an ongoing series of posts about the medical industrial complex is well worth the time and effort.

      And since my guess is that your readership is older, sicker, and savvier (this ain’t our first rigged rodeo), we’ll be able to contribute plenty of scams seen and insidery tricks learned.

      (Tip #1 – Been told your desired doc – or class of Drs. (rheumatologists!) – don’t have any slots open for 6 months to a year?! (Happened in Vegas, baby)…**call various offices daily/weekly for that day’s/week’s cancellation slots** Perhaps obvious in retrospect, but it took 6+ calls to different Drs., to be clued in by office personnel.)

      I also greatly appreciated Mike not letting the medical personnel skate regarding the chaos in the US med system.

      (Having a doc say he isn’t involved in the billing process is a bit like saying he doesn’t pay attention to the “getting paid process”…you can get bet your sweet bippie that Drs. don’t net $150k to $300k per year (medians) by not paying attention to financial matters. But most commentators treat med personnel in the med industrial complex like innocent babes in a war zone).

      Look forward to future posts, Mike.

      • Korvash says:

        Here’s a tip for all:
        Do not count on your doctor, or heath plan or Wall Street conjob having, or providing to you when you need them with your medical records. Systems crash, things get lost, people are careless.

        Get a hard copy, on a disk of every X-Ray, Cat Scan, MRI, or paper of lab tests and examinations.
        In California they have to give it to you for free at the time of service. Demand it before the procedure, that way it’s generated at the time. If it’ll take 20 minutes to make the disk, come back for it.

        Write in ink, on your agreement, to pay which you have to sign before service, “Bill only paid after hard copies of exam results delivered.”

        Had a provider fail to deliver a disk because “their machine was broken.” Waited and waited for weeks. Refused to pay.
        Went to collection.

        “Read the contractual agreement to pay sheet”.
        They stopped calling.

        Treat the institutional enemy with no mercy. That’s all they deserve. Be kind to people, it’s rarely their fault.

        • Dr Dearth says:

          The institutional machinery was built to serve business, not people, on purpose. We can tear down and build something that actually serves us.

      • Paddy Jim Baggot MD says:

        DOCTORS ARE MORE THE VICTIMS THAN THE PRACTITIONERS OF SCAM

        • Wolf Richter says:

          Paddy Jim Baggot MD,

          Please locate the CapsLock key on your keyboard (usually on the left somewhere) and press it once. This will allow you to comment in lower-case. I don’t allow all-caps comments. Thank you.

        • Paddy Jim Baggot MD says:

          sorry about all caps comments
          i have great respect for you, wolf
          i have been reading you for years
          you are one of the most insightful authors

    • cdr says:

      My last physical, 2 years ago, was bizarre. The Dr spent all his time typing into a tablet computer. Almost feverishly. He asked almost no questions. Every time I said something about anything, type type type.

      The Medicare required questions were humiliating and infuriating. I failed the memory test because my memory has Always worked differently than most. My short term memory is poor and always has been unless I am interested in what I am supposed to remember. Then, it’s like an elephant’s memory. Someone telling me that ‘I need to remember this’ makes it important to them, not me. I intentionally failed the clock test and said ‘I forget’ on the other part. When needed, I take notes in real life to deal with names and such, which I also rarely care about. Not looking forward to another physical someday.

      • cdr says:

        To add one more point:

        The thing that rankles me the most wasn’t mentioned above.

        As soon as the office nurse mentioned the memory test, I knew I would do poorly on it by their standards. My memory has worked contrary to their standards for my entire life, yet I have somehow muddled through and been successful.

        I mentioned this to the office nurse and asked if we could skip the memory test. It’s humiliating to explain myself like this.

        Her insipid reply contained an implied threat that Medicare would be very upset with me if I gave her any problems over this test. I asked her what Medicare would do if my answer was ‘I Forgot’? She mumbled another implied threat that included the word Medicare.

        My reply was “I Forgot.”

        Then, on the clock test, I reversed the big and little hands on the face of the clock.

        • Nick Kelly says:

          My mother at 92 had a mem test where they asked her to give alphabet backwards for 5-6 letters. I can’t do it, never have been able to.

    • Nolan Squire says:

      Bravo – thanks for taking the time and putting in the effort to be a “blade of grass” fighting the malignancy that is corporate medicine. We more than ever need medicine to be just that, a healing force that helps us live and be healthier – even if it comes with a bitter taste.

      Looking forward to reading your work.

    • Moosy says:

      Yup.

      The DMV is now a sunny easy breeze compared to the insanity of healthcare.

      It’s what happens when things are offered for free. Always some hidden negatives

    • david ward says:

      My advise is for you to watch the documentary:
      “Quest for the Cure..Ep 1, The Cancer Pandemic”
      on utube

    • raxadian says:

      There was a recent article on how is cheaper if you do not have Health insurance unless said heath insurance is a state one like Medicare.

  2. ivanislav says:

    Great article. I was on the wrong end of fraudulent/over- billing by a major healthcare provider, but at least it never went to collections and they adjusted the bill after I called them on it. By law the cost of service should be provided up front for all but life-and-death situations, as far as I’m concerned.

    • Michael Gorback says:

      It’s only very recently that cost of care has to be disclosed and it’s mostly for hospitals.

      “Surprise billing” as it’s referred to is just now being addressed.

      FYI in Texas there is legal precedent that you can contest a hospital bill by demanding they open their books to reveal their usual payment for a given service. I think the principle could be applied in any situation but the legal costs are formidable.

      Usually it takes a large painful judgment to get a lawyer in the game.

      In my practice we checked insurance benefits, how much deductible had been met, co-pays, etc, and gave the patient our best estimate of cost. We only did this for high-ticket services, not for every visit. That would be cost-prohibitive.

      Now you can ask a hospital for full disclosure of costs. This is new!

      • Otishertz says:

        I’ve ended up in the emergency rooms a couple of times in the last decade for a broken hand and some stitches. When they handed me the financial agreement form I asked for a sharpie and wrote on it:

        “All charges not agreed to in advance will not be paid. I cannot agree to pay a bill if I do not know what it will be.”

        Then I got the clerk to initial it. Guess what? They got nothing from me. They forgave all the balances as “charity” when I showed them the agreement they signed. I guess it wasn’t worth arguing with me. I could afford to pay, too. Such was/is my contempt for US medical services.

        There is no excuse for them to not know what to charge for five stitches or a simple bone fracture in the emergency room, except shameless greed.

        I also took the sharpie and wrote a big X on the privacy policy and the word, “NO.” I’m that guy, hahaha.

        I got the added bonus of contracting the 2009 H1N1 influenza in the emergency room, was horribly sick for two weeks.

        • Dr. J says:

          While I sympathize with the sentiment, sometimes it’s hard to know “up front” to the penny what the cost is, similar to construction when they removed perfectly good drywall and notice the back is covered in black mold. In construction, they usually allow for this contingency. Same with a car…”hey chief, got a small leak at the water pump” and it turns out you got a cracked head leaking into the oil pan. Again, you can stop the work, but amazing the complaints generated against some one-man-band mechanic just trying to pay the bills. I’ve not seen the “Trump List” at the local hospitals (supposedly breaks down costs for a consumer to compare hospitals), but as a provider, we tell the patient the up front cost of most common procedures. The way doctors set up fee schedules it to find the highest cost your best insurance company reimburses and add 5-50 bucks to that. It does come down sometimes to a “stroke of the chin whiskers” for small practitioners. Corporate medicine has already gamed the system since “they have people for that”. The guy who gets hammered is the one with no insurance, unless you came to my office. We discounted often, depending on circumstances. My fav…”do you give a senior discount?” My response…”do you need it?”. If no, then I say, what you just paid here helps us help a mom with 4 kids who had a husband who went out for a pack of smokes 10 years ago and never came back.” Never lost a patient over this policy. Again, thanks for letting me vent.

        • Otishertz says:

          Forgot to mention, I was excluded from the medical/insurance racket for most of my adult due to a condition I contracted in a hospital at age 13.

          I have nothing but contempt for the medical services system and saw it as my duty to resist and to not pay them one red cent.

          I cringe when people use terms like “health care” and “provider”. I cringe when I see health care companies naming stadiums. I cringe when I see huge light-up crucifixes emblazoned on the front of colossal medical edifices.

          Why do so many of them have the symbol of torture and death of a savior front and center?

        • Michael Gorback says:

          A little over the top with the hate but I understand your anger. However, you can’t lump everyone into this. The biggest problem as I see it is that corporations control medicine now. Well-meaning doctors who object to how things are done are tagged as troublemakers.

          It’s the doctors’ fault. We were so busy squabbling amongst ourselves that we lost control. It was a circular firing squad.

          For instance, Medicare uses RVUs – relative value units – to determine payments. There’s a committee representing the specialties that assigns RVUs. So the specialties fought over these because Medicare says “This is the size of the pie”. So we fought over our pie slices instead of saying “You’re gonna need a bigger pie”.

          Then along came HMOs. You had to be on their “panel” or you couldn’t see their patients. So (hypothetically) Aetna offers you a contract. You don’t like the contract and try to negotiate. Aetna says no contract, no access to our 10,000 patients in this area.

          David vs Goliath but no slingshot.

          Independent practices can’t share contract info. FTC regs. Somehow the insurers know our contracts. Search for “silent PPO”. I’d go into it but it’s not something that will help patients much. More of a nasty way of cutting fees.

          The worst development recently has been private equity getting into the game. I think none of us believe “health care” and “private equity” belong in the same sentence or even the same dictionary.

      • wkevinw says:

        I am pretty sure this could be significantly improved (the pain and high cost of medical care in the US), by a team of professionals from the medical, business and insurance side of things. There would have to be motivation, however, for it to be done with any kind of diligence. The fact that it still hasn’t been done is very telling. None of these organizations wants to do it. The regulators have been captured, costs socialized, etc. There would have to be more competition, with price discovery, etc. The government would have to come up with skillful regulation.

        It is actually one of the things bankrupting the country; because a vast majority of people believe “medical care is a right!”- which it isn’t.

        Medical care needs sensible profitability brought on by competitive free enterprise. In addition there has to be a mechanism to provide care that is within the ethical boundary to those who cannot immediately afford it. It could be billed after the care as an attachment to wages/collected by some government entity.

        This problem could be “solved”.

  3. Questa Nota says:

    Always save receipts and review bills, for medicine or other services or goods. There are so many opportunities for mistakes to creep into the system, by design, dark pattern website manipulation, lack of training or other factors. Expect that any private equity involvement will include some billing breakage factor where the cost of the error is borne by the patient and the benefit accrues to the provider. They count on uncontested billing and acquiescent clients. The same holds true for financial services.

    Caveat Emptor has never been more applicable.

    • Harrold says:

      Your not kidding!

      My elderly mother had a surgery, of which the total cost was paid for by her insurance & medicare.

      A month later she received a bill for a little bit over a thousand dollars. She called the questioned them about this charge and the billing person agreed it was just a mistake and to throw it away. The following month she received the same bill and again called and they agreed it was a mistake. At receiving the 3rd bill, she realized it was an attempt scam her out of her money. She continued to receive bills for the next 6 months.

      • Michael Gorback says:

        Re-read my OP. Tell the billing people you’re sending this to the OIG (Office of the Inspector General) at Medicare.

        Sometimes it’s hard to determine if it’s stupidity, a bad billing system, or malice but enough is enough and you kick it to the authorities.

  4. Steal $400 with a gun and you get stuck in a cage for life.
    Steal $400,000,000 with a pen and you probably just get a fine, or at worst case, a few years in a country club with fences.

    Wall Street types have figured this out a long time ago. Most likely they’ll eventually get hit with lawsuits or fines, which are just a cost of doing business. Theft should be punished as theft, regardless of whether a gun or a pen is used…

    • El Katz says:

      They get hit with fines or lawsuits, then raise “fees” and prices to compensate themselves.

      The fines should be paid through prior and future bonuses for execs. A nice clawback a few times could be a motivator – especially when they have to sell the manse and the yacht to pay it off.

      • Nathan Dumbrowski says:

        Agree. Gives are just fees under the guise of penalties. The bonus is untouchable for the most part.

    • Wisdom Seeker says:

      Steal $400,000,000 with a pen and you hire lobbyists, make campaign contributions, maybe even buy yourself a seat in Congress.

      “Behind every great fortune is a great crime.”

      (Note: some crimes have been legalized by the criminals. Why again is Congress not subject to the same laws as everyone else?)

    • Michael Gorback says:

      “A lawyer with his briefcase can steal more than a hundred men with guns.” Vito Corleone

      • Catxman says:

        This is similar to the proverb:

        A man with $100 owing the bank has a problem.

        A man with $100 million owing the bank : bank has a big problem.

        • NBay says:

          If a monkey hogs all the bananas and the others get none, scientists would study him to see what is wrong with him.
          If a human does it they put him on the cover of Forbes.

        • Michael Gorback says:

          Was that Donald Trump who said that? ; -)

          Formula:

          Fail so spectacularly that you might take down the NY banking system.

          Cut a deal.

          Write a book called The Art of the Comeback.

      • Nick Kelly says:

        Trump has never written a book and likely never read one, although he may have skimmed Art of the Deal by Tony Schwartz. Tony has apologized for his role in creating the T myth ‘I needed the money’ and it came with a rare 50/50 split. Even at that he almost backed out: ‘he wouldn’t sit and talk for 10 minutes’
        The project proceeded when T let Tony listen in on phone calls. That is when Tony discovered that T routinely eavesdropped on guest calls.

      • Paddy Jim Baggot MD says:

        THANK YOU FOR DOING THIS
        IM A DOCTOR
        I STILL NEED TO LEARN

  5. Nick Kelly says:

    Good stuff, but little puzzled by why EMR is bad. How often when some celeb ODs they had been doctor shopping and getting multiple prescriptions.
    Suppose a patient is admitted unconscious. How does the ER know what he’s allergic to, or preexisting conditions. Are they to wait until they get hold of his doc to look thru his paper records?

    However the crux of the overbilling problem is that this doc is dealing with the most expensive and most dis-functional system in the G7.

    • Cynical Engineer says:

      EMR is a great idea which has been hobbled by a horrible implementation.

      First problem: There is no central EMR system. Each provider has their own EMR system which is rarely interconnected to any other provider. When you roll into that ER, they know nothing more about you today than they did pre-EMR.

      When you switch providers, even if you take a copy of your records from your old provider to the new, it’s a crapshoot if they’ll be able to import your old records into their system in a meaningful way.

      The second problem: In order to try and provide consistency in records and billing, everything a medical provider does must be “coded.” There’s something like 30,000 of these codes and they are revised every few years. Frequently, figuring out the correct code takes longer than it took to provide the original service.

      That leads to the current situation where a single doctor typically requires SIX full-time office staff in order to handle the records management, insurance billing, patient billing and more. That’s a LOT of mouths to feed.

      • Cas127 says:

        “Each provider has their own EMR system which is rarely interconnected to any other provider.”

        In general, I agree with you. But when *any* industry complains about the “impossibility” of file sharing/interoperability, I immediately think “ASCII and print spooler files”.

        Almost any software can output ASCII files, which in turn can be read by most software (definitely databases, which are the guts of most systems).

        Ditto print output/spooler files, which are frequently available in maximally stripped down versions, in order to ensure absolute maximum interoperability with *printers*.

        It may not be elegant or smooth to schlep ASCII files from database A to database B, or have to cross reference the database fields…but it is far from impossible.

        Even more so if somebody/anybody on the front end makes any effort to weigh interoperability at the outset.

        But my sense is that historically (post 1965) medicine has been so un-cost constrained, lucrative an industry, that bad business and financial practices have been able to survive long after other industries fixed them.

        The “chaos costs” just got passed on to patients and, especially, gormless government.

        • Michael J Puglin says:

          There are well-known, standard data exchange formats for EMR and EHR. There is little excuse for a lack of interoperability.

          Trying to schlep ascii files would be a pointless exercise in futility.

        • Michael Gorback says:

          And yet a decade later and there’s no interconnectivity.

      • Harry Houndstooth says:

        True Wisdom

        EMRs (Electronic Medical Records)
        A uniform standard should have been set from the beginning insuring all EMRs could communicate universally. Software business must try the opposite: trap the user into a unending stream of revenue.

        It’s just business.

        • Cas127 says:

          HH,

          I agree, software makers would prefer to trap users into proprietary solutions so they can never leave.

          But,

          1) A small amount of front end research reveals the worst abusers,

          2) Almost all software allows some variation of an ASCII export,

          3) Even without ASCII export, interoperability with things like printers, etc. usually means having to create stripped down files that can be ferreted out and used to export.

          The whole point of my first post was that medical software users don’t *have to* remain prisoners of walled gardens that make a mockery of electronic file sharing across the medical industrial complex.

          Hell, my guess is that there are software packages/consultants who will facilitate these sorts of things (there are plenty in other industries).

          But…another guess is that the MIC really doesn’t *want* freely flowing patient info…since it promotes competition among medical *providers*.

          And they *love* there own walled gardens (see Certificate of “Need”, etc)

        • Paddy Jim Baggot MD says:

          A PAPER CHART IS NOT DESIGNED TO GET INFORMATION OUT.
          ITS DESIGNED TO PUT INFORMATION IN..
          emrS ARE FAR WORSE.

          EMRS ARE EXTREMELY CUMBERSOME. OFTENA A PATIENT EVALUATION IS TWENTY PAGES. RELEVANT TO THE REASON THE PATIENT CAME TO THE DOCTOR, SCATTERED PHRASES HERE AND THERE THROUGHOUT THE CHART. THE IMPORTANT INFORMATION IS HIDDEN LIKE A NEEDLE IN A HAYSTACK.

        • Wolf Richter says:

          Paddy Jim Baggot MD,

          Second and last warning: don’t use ALLCAPS. All your future comments in ALLCAPS will be deleted.

    • Lisa says:

      EMR reduces the patient doctor time as the dr has to spend more time with documentation. It takes away from the patient. It is data collection

      • Cas127 says:

        Lisa,

        Maybe…but I’ve had Drs. absolutely refuse to take/read my pre-prepared detailed medical history…they absolutely insist upon wholesale verbal recapitulation, which they then insist upon slooowly transcribing themselves. (The only place I’ve seen dedicated medical scribes is in ICUs).

        I basically assume this is CYA for possible future legal liability (ie, ain’t nuttin going into their legally discoverable files that they didn’t write themselves…) but it slows things waaayyy the hell down, unnecessarily.

        *And* it contributes to errors…since I know my med history best and can take hours and hours paring it down accurately. Not something a rushed Dr. can do effectively in a 20 minute appt).

      • Paddy Jim Baggot MD says:

        THE EMR IS VERY CUMBERSOME TO COMPLETE. ONE WOULD SOMETIMES BE FACED WITH THE CHOICE TO EITHER CHART ACCURATELY AND HONESTLY, OR TO MAKE UP bs AND LEAVE OUT THINGS WHICH ARE IMPORTANT.
        THE PENALTY FOR BEING ACCURATE AND HONEST IS AN EXTRA HALD HOUR OF UNCOM,PENSATED TYPING.

        SO PEOPLE HAVE AN INCENTIVE TO PUT NONSENSE IN THE CHART. AND THEY THEREFORE DO.
        THIS IS ONE REASON WHY EMR IS SO USELESS

    • Nick Kelly says:

      PS: read a book while back by a doc, black guy if that narrows it down, that is mainly about a more holistic approach but he also noted the typical MD spends 1-2 hrs per day talking / negotiating with insurance cos. This is the most expensive time in the system. Harvard did a study on US meds and noted EVERYTHING is the most expensive in the world. It also concluded it wastes 200 billion per year.

      The proof of the pudding is in the eating. 6 out of 7 G7 countries can’t all be wrong. There is no longer any doubt that a single payer, universal insurance plan works best. In the case of Canada it covers most things for everyone at about half the cost per person.

      • NBay says:

        Agree 100%.

      • Harry Houndstooth says:

        Approaching 20% of GDP, we Americans can no longer afford our system of health care financing. Our health continues to get worse and worse. Corporations are not interesting in lowering the cost of healthcare and improving the health of Americans. They want the opposite.

        It’s just business.

    • Raging Texan says:

      EMR’s would be fine, there’s nothing wrong about the idea of using electronic records.

      It’s the implementation, regulations and the fedgov protected oligopolies set up to provide them to providers, etc

      Because Americans don’t know their own laws, they would have allowed for and even demanded the fedgov to mandate an equally ridiculous and productivity destroying paper system, but by making them electronic it provides more billing opportunities for “businesses” that lobby the fedgov.

    • Michael Gorback says:

      EMR is a burdensome task. Think about how much time is spent clicking on drop down menus alone. Most doctors spend hours per day (often after the schedule has been cleared) doing their EMR homework.

      EMR is also a great way to “build the bill”. You can pull in all the previous info, including notes, and generate a new note. I do reviews of personal injury cases and it’s not uncommon to be able to hold two office notes up to the light and superimpose them word for word.

      So I might see you for 2 minutes and then pull in all the info from a more detailed encounter, then upcharge you.

      The notes are usually pre-populated and you click on anything you want to edit. So the boilerplate might say “neck supple” but in the physical exam further down where they document the musculoskeletal exam it says “range of motion decreased in all planes limited by pain and tenderness”.

      In all my years of practice anything worthwhile was welcomed. You didn’t have to force us to adopt new things. We wanted them. The healthcare community was led into EMR by government coercion and false promises.

      • taxpayer says:

        For a couple years the place where I am a patient would have a “scribe” accompanying the MD at each exam, with a laptop, taking notes, which seemed to free up the MD to actually talk to the patient. I don’t know what their training was. More recently, the scribes are gone. Obviously they cost money but seemed to facilitate treatment.

    • Michael Gorback says:

      The failure to implement the unicorn promise of interconnectivity means the ER can’t look that up.

      There are also privacy rules (HIPAA) that prevent anyone from accessing your health info without permission even if those unfulfilled BS promises had borne fruit.

      But you can catch doctor-shopping easily because most (if not all) states now now have a prescription monitoring program where you can look up a patient’s prescription record for scheduled drugs.

      If you abuse the database by looking up prescriptions for controlled substances on someone who isn’t your patient you can get into trouble. I have never heard of that happening. Either everyone is scared, everyone is ethical, or there’s no enforcement. Guess which is most likely?

      I saw my family doctor today and he actually checked my prescription records!

  6. cb says:

    Hey, if a career doesn’t work out with the thieving FED there’s always the medical/industrial/insurance healthscam industry.

  7. jimq says:

    Been there, done that!

  8. JAS says:

    UK resident. NHS. Have none of these problems as care is paid for by general taxation. No it isn’t socialist it’s the way we do it. I feel for you guys I really do.

    • stan65 says:

      U.K. resident. NHS.

      We don’t get the financial ripoff that our yank cousins do, but we get it in the neck with the care lottery that NHS has become over last 10-20 years.

      What that?

      Well you can find yourself in a NHS hospital thinking that you are receiving the world class medical expertise that BBC say NHS delivers. But no, you get what is served to you on the day, ie inexperienced doctors, trial-and-error surgeons, illiterate ward staff who can’t tell chemotherapy from blood transfusion.

      If you do not know what is being done to you in our deified NHS, then, it’s lottery time.

      If you care to learn about your ailment, there is a modest chance of walking out not crippled or disabled.

      • Whatsthepoint says:

        Absolutely….I’ve been in that carousel many times with my husband…incompetent anesthetist that almost killed him…nurses giving medication his records stated he was allergic to….loads of ‘docsplaining ‘ and bickering. Bottom line, you’d better learn to be a good self advocate and get educated. Nonetheless it’s better than going broke and I’d take it any day over the US system….and I’ve seen that up close and personal, too.

        • Paddy Jim Baggot MD says:

          i met one patient who was a nurse. She had an orthopedic surgery at an NHS hospital. She was in excruciating pain all night long. She complained to the nurses many times but they never call the doctor, who did not come till the next morning. He found her hip had been dislocated all night long and that was why she was in so much pain.
          after that experience, she still maintained the NHS was the greatest in the world.

      • Augustus Frost says:

        Advocates of universal “free” medical care prefer to overlook the negatives.

        The first one is that there are supposedly no supply constraints and demand won’t outstrip it because it doesn’t matter if someone doesn’t pay the bill themselves. Never any rationing in universal government care, right?

        Second, since apparently everyone has a birthright to unlimited medical care at someone else’s expense, everyone is also entitled to live as they please, the consequences be dammed. Drug addiction, bad diet, and a sedimentary life style? No problem.

        At some point in the future, the government is going to start telling people what they can and can’t do with their life, explicitly. Open ended entitlements, aging demographics, and open borders do not equal solvent governments and sustainable social programs.

        I know the US medical system is a disaster but doesn’t change the math for either.

        Good luck

        • Apple says:

          The government has always been telling women what they can and can’t do.

          You’d be amazed the hoops and cost to get something simple like birth control pills.

        • Nick Kelly says:

          ‘Second, since apparently everyone has a birthright to unlimited medical care at someone else’s expense, everyone is also entitled to live as they please, the consequences be dammed. Drug addiction, bad diet, and a sedimentary life style? No problem.’

          Again the proof is in. Universal single payer has been in Canada for 50 + years. Like all the affluent West it has lifestyle issues but if anything the US is worse. l

          Now we’ve disagreed let’s have a laugh about a typo. We all so them WR has some classics. You refer to a ‘sedimentary life style. Would this be a cave dweller?

        • Whatsthepoint says:

          No one is overlooking negatives….they exist but can be mitigated with education and self advocacy (being a loud mouthed, assertive b!@^h)…still better than being skinned and eaten alive by marauding piranhas…

      • JoeC says:

        Come visit any rural area in the US. There are 10 terrible hospitals for every good hospital in the US. And I have worked at some of the top hospitals. Have a rare disease, they are great. Something run of the mill? The doctors are bored and you get less quality care than expected.

    • Michael J Puglin says:

      It isn’t socialist? Come again?

      • JAS says:

        WE have had more right wing governments than socialist governments since 1900. And no socialist government for years. And yet curiously the NHS keeps on going. I think the US interpretation of socialist is a lot different to the UK version. I’d never vote for a socialist government in a million years but I vote for the NHS every time.

      • Nick Kelly says:

        In Canada the medical INSURANCE is socialist, in that it is funded by gov. Don’t you drive on gov funded socialist hiways ?

        The doctors are in private business. Most specialists are incorporated.
        It’s a hybrid system. The debate is over about which is best. The ‘experiments’ have been with the populations of the G6 over a period of half a century.

        BTW: I don’t know that much about the US system, but one question. Is it possible some insurers are profiting by doing a kind of wholesale / retail biz? Like buying procedures, supplies etc. and marking them up?

    • tfourier says:

      @jas

      And you pay almost 15% / 20% of you income tax to pay for the NHS. If you make any sort of reasonable income. That 120 billion plus quid (2019) has to be paid somehow. And if you want decent treatment and not be put on a waiting list or play MRSA lottery you have to go private.

      Then like all socialized systems there is no excess capacity so it is on the verge of collapse every flu season and completely collapses during a crisis. Like it did last year. Just like the health care systems in Italy and Spain. But not France, Netherlands, Switzerland or Germany. See the pattern?

      Having dealt with the NHS and the US system I’ll take the US system any day. The Dutch, Swiss and even French systems might be cheaper and less dysfunctional than the US system but I have found that the moment you mention the magic words “Self Pay” the US system can be quite inexpensive and the quality of treatment is second to none. Based on many decades of personal experience with long term health problems.

      The US system is dysfunctional because it is the only country in the world where heath insurance is tied to employment and the only reason its that way is because of Organized Labor. What started as a way of getting around war time wage controls in the 1940’s has metastasized into an unreformable train wreck. Break that link with employment and watch the system quickly regain some kind of sanity.

      And Medicare as it currently exists is just a way of the wealthiest age cohort who account for 80% plus of all health service usage have their health care costs subsidized by everyone else. The waste of this system is truly staggering. Replace with a French or Dutch style health insurance system and watch the costs collapse.

  9. Petunia says:

    Interested in your views on universal coverage and those of your profession in general. I don’t begrudge a professional a good living, but what we have is obscene price gouging.

    Currently waiting for a bill to be review by the insurance company. We were charged full retail by a provider because they didn’t get the pre-approval they knew was needed. The treatment would have been covered and discounted to the scheduled amount had they done it correctly. Instead we had to pay the full amount up front to get treatment.

    • Michael Gorback says:

      Doctors are making good money but not on the fees. We learned to invest in the corporate world that has effective lobbyists. Hospitals compete tooth and nail, but when they have to fight the government they become a Viking shield wall.

      So we invested in compounding pharmacies, ambulatory centers, MRI facilities, urine drug testing labs. I made more money on those than my practice. The grand slam was building our own hospital.

      Guess what? The big box hospitals did everything they could to kill us. Many physician-owned hospitals were strangled in their cribs by bureaucrats and politicians. Insurance companies were told if they gave us contracts the big box hospitals would cancel their contracts. These boys play rough.

      Some got bought out. When I was chairman of the board at our hospital the CEO of a major hospital chain called me about buying us out. We’d only been open a year. That’s how much money there is in hospitals.

      Congress passed legislation banning doctors from building new hospitals and existing hospitals were banned from expanding beds.

      Sometimes I think if I could do it all over again I’d just build hospitals and ambulatory surgery centers.

      • NARmageddon says:

        >>Congress passed legislation banning doctors from building new hospitals and existing hospitals were banned from expanding beds.

        Which law and year was that?

      • lenert says:

        How about building a few more medical schools? Oh, wait, no money in it and we just end up with more doctors serving more patients for less money.

      • Paddy Jim Baggot MD says:

        right. all money goes to hospitals and insurance companies. Both exploit doctors and patients shamefully.
        i had a patient who ruptured membranes at 24 weeks.
        baby mortality about 50% or more. If it survives, 3-4 months in NICU if everything goes well. Many complications, short term and long term likely.
        I kept her undelivered until 29 weeks. intact survival better than 90. No major complications.
        for all this momentous responsibility, i get paid $30/day for inpatient visit. Nurses get 40/hr x 12 hrs = 480/shift, 960/24hrs.
        nurses paid way better than doctors
        hospital paid way better than nurses.
        for preventing disaster, i get paid almost nothing. $30 would not cover cab ride to hospital.

        for most hospital; bills, all doctors together get paid less than 5% of total bill.

  10. billytrip says:

    Anyone that has not figured out that the health care system in the US is a carefully organized system for sucking every possible dollar from the patient’s wallet is not paying attention or not too bright.

    There is no such thing as “free market” forces for services and products that are rife with monopoly interests and not discretionary purchases to being with.

    Single payer is the only answer that will work but the “socialism” morons will prevent that from every happening. Good luck to all of you who ever have serious health problems.

    • Worker Shrugged says:

      WELL SAID! From opaque pricing, very constrained supply of physicians, to many monopolized local hospital markets, VERY LITTLE about our medical system is Free Market regarding supply -and unaffordable to a majority of patients now.

      When a doctor friend told me he want the government out of medicine and told him this means NO medical license from the government…..

      he said he means out of everything else EXCEPT licensing laws LOL

    • Michael Gorback says:

      A free market could work. Doctors are very competitive. One might argue that before HMOs and all the other stuff we could have had a free market. In a sense yes, but a free market requires price discovery. Back in the day a typical insurance plan picked up maybe 80% of the cost no matter what. You paid the doctor and submitted the bill to your insurer. The doctor had NO overhead for a billing specialist, collections specialist, software, etc. Helped keep costs down.

      But the “80% of whatever was charged” system got out of control and we got HMOs and drive-by childbirth. Remember Helen Hunt’ rant about HMOs in “As Good as It Gets”? Audiences literally cheered. I read a news article about how HMO execs in the theaters had no idea how hated they were.

      Insurance insulates the patient from the true price. Undoing this system is like trying to put the toothpaste back in the tube.

      Whether it’s the NHS, the Canadian system, or whatever, I can tell you the only happy customers are the healthy ones who haven’t had to have serious care. Most have parallel public/private systems. Do you think Richard Branson uses the NHS?

      And someone, somewhere, pays for it no matter what the system is.

      • Jim Mitchel says:

        Price discovery? Cars have window stickers and funeral directors have price lists. Car mechanics have binding estimates. All are required by law. Doctors and dentists have none of the above. When Little Fluffy at the doctor’s front desk won’t tell me if a procedure is paid under medicare, it makes the doctor look like a crook.

        • Michael Gorback says:

          I just paid for my mother’s cremation a few months ago. That’s when you’re really vulnerable. $4500.

          I told my kids to cremate me and flush me down the toilet. Who needs an effing urn?

          They were appalled. I told them I’d eventually make it it into the ocean and with any luck some of my molecules would end up in Melania Trump’s nether regions in Tahiti.

          Children these days have no sense of humor.

      • Say It Aint So says:

        Great article…Thank You…at the end of the day the population is controlled by Big Insurance…Big tech…and Big Pharma…the 3 largest Lobbying groups…Hospitals are more and more turning people over to Collections…when I called the Hospital and asked them to resubmit the bill they said it is the “Health System’s policy to only submit a bill one time then off to collections!”…The premiums we are paying is criminal and the Insurance Companies continue to report Billion dollar QUARTERLY profits…They knew this in 2010 when Obama rammed this down our throats…This is on Obama and his cronies…You dont get rewarded with $14 million dollar “summer home” on Martha’s Vineyard for nothing..

      • Nick Kelly says:

        1. Few years back my wife had shortness of breath at 2 AM. Ambulance called. After nite in local hospital (Van Isle) she is whisked to Royal Jubilee in Victoria. After one nite she has stent put in near heart, via wrist. After one more nite I go pick her up.
        So far no charge–zero. Next week she’s back at desk at paper.

        2. Her brother was in a toboggan accident. (40 yrs old) his foot was
        twisted until facing backwards. Operated on by top man, with steel
        pins all over. Fine now. No charge for op.

        3. I burnt arm in house fire. Just red so I ignored. Then it bubbled up. After a nurse friend kind of freaked out I went to ER. Doc on duty freaks out, puts me on drip (1st time in life) makes a call comes back and says ‘don’t worry you’ll be seen by a top man’, For the fist time I think jees. So that nite I see specialist who doesn’t freak out.
        Funny part of treatment was going to have nurse pick dead skin off arm every day or so. No charge.

        4. Friends of family on Van Isle had VERY premie baby delivered ( under 2 lbs) Helicopter arrives from Vancouver with premie specialist team aboard. Kid fine now. All covered

        An example of the massive US confusion occurred when US Senator Paul traveled to the Shouldice Clinic in Canada for shoulder surgery.
        Typical comment in answer to question: ‘how can this socialist system be preferred by a US Senator’

        ‘It’s a private clinic’ implying that the average Canadian can’t go there and be covered.

        Of course it’s a private clinic, meaning privately owned, and it treats average Canadians all the time, by law. The Senator, who of course wasn’t covered went for the quality, although given outrageous US bills be might have saved money also.

        I do know the UK has two systems. Canada doesn’t. I don’t write about the NHS because I don’t know about it.

        • Nick Kelly says:

          Of course there is no free lunch or health care. But one very hungry player has been eliminated: the insurance complex which never sees a patient. Stories of negotiating with them, or offering cash to get lower bill, etc. are non-existent in Canadian medicine.

          They exist in dentistry however. Whenever I go I say: I’m not on a plan. ‘Got ya’ they say and it makes a difference.

  11. Pavel says:

    Don’t forget the outrageous mark-ups for aspirin, bandages, saline etc… There is massive overcharging at every level.

    A friend had a wedding in Venice a few years back. Her 85 year old uncle had a mild cardiac event and was taken to one of the hospitals where he spent three days, had multiple scans and constant monitoring, and was then discharged. The total bill was something like $500.

    • VintageVNvet says:

      Please be clear P, that you are referring to Venice, Italy!!
      Friend had zero BP a couple years ago, went to ER about a mile away, was ”admitted” overnight, given IV saline and then aspirin; bill was $80,000.00!!!
      She was lucky her hubby had insisted on getting two ”supplementary” insurances besides her medicare, so only a hundred or so out of pocket..
      CRAZY CRAZY CRAZY system of medical services delivery in USA,,, thats for damn shore!

      • Pavel says:

        It was indeed Venice, Italy, and the most spectacular wedding in the world :)

        Sorry to hear about your friend’s experience, which sadly is not that unusual in the USA.

    • Nick Kelly says:

      Insulin is 1000% of the Canadian price.

      • Michael Gorback says:

        In Canada hospitals are “free”, doctors are “free”, but unless something has changed medication is not covered. Price discovery.

        Some Canadians have medication insurance.

        I don’t have time to fact check the insulin prices but it seems kind of off.

        The funny-yet-sad part is Biden wants to allow US citizens to buy meds from Canada. I’ll bet some of those cheaper meds are made in the US.

        Years ago an orthopedic group discovered that SynVisc was cheaper in Europe than the US. They started buying Synvisc overseas. Synvisc is manufactured in the US. The orthopedic group was nailed by the Feds for reimporting drugs.

        • Apple says:

          Canada charges ~$40 per vial of insulin.

          In the US that same vial of insulin is ~$300.

          Insulin was invented 100 years ago.

        • Sams says:

          Not sure about Canada, but the Canadian health deparment may negotiate the price as single buyer for all prescribed medicine.

        • Island Teal says:

          Synvisic…you said Synvisic 😬😬
          We should talk about that 7 or 8% of the patients that suffered with the pain of a negative reaction. I know. I was one of them 😬😬

    • Michael Gorback says:

      It’s called the chargemaster. An imaginary list of outrageous prices that only the uninsured pay.

      My wife had an appendectomy 20 years ago. The bill was $80,000. After contractual writeoffs by insurance, $2,000.

      Always get that EOB before you pay a Nicole. Meducare can send you an MSN.

  12. Ron says:

    Great information, please give us more.

  13. Al Loco says:

    I noticed a difference with EMR. Most equate computers with efficiency but it’s just not the case. Just like an ERP, it’s all about implementation and if not done well, everything goes to $h!t.

  14. Sam says:

    Doc Gorback,

    “In God we trust, all others cash” has served me well with a high level of care (from MD’s, ND’s & DDS’s) as “money does not talk, it swears” (ie Bob Dylan). Especially with Flight Doc’s and their “A” lists.
    Thank You for your honesty, insight, & integrity. May your professional journeys be of minimal headwinds & turbulence.

    re Questa Nota: “Caveat Emptor” – True!

    “…among the calamities of war may be jointly numbered the diminution of the love of truth, by the falsehoods which interest dictates and credulity encourages.” Source: “The Idler” magazine from 11/11/1758.

    • Michael Gorback says:

      You can try to negotiateca cash price and it often works. My patients would complain that the MRI I ordered was going to cost $1500 with insurance. I told them to offer $450 cash. No problem. I think Medicare pays less than $300 so $400 looks good.

      I used to publish my cash prices online at directpaypain.com. I abandoned the site years ago but whoever owns it won’t take it down. If it’s still up you can see that I was using Medicare rates plus 5% and the date the rates were set was about 6 or 7 years ago. By the time I closed my practice I was just charging Medicare rates. Nobody else in my specialty in the community accepted cash and I think that’s still the case.

      Sometimes I did things for free if it was an established patient who lost their job and insurance. At Xmas we’d go through the outstanding bills and if we found someone who we knew was living on a shoestring but was diligently sending in $10 every month on a $500 balance, we’d just write it off as a Xmas present.

      I wasn’t the only doc who did that. I got the idea from someone else. Sometimes you stand tallest when you stoop to pick up someone else. I’m only 5’7″ so I need some help on the height thing.

      • lenert says:

        In the documentary “Healthcare Around the World” T.R. Reid reported that in Japan an MRI costs thirteen dollars.

  15. marc lippman says:

    this article makes a good point poorly. there is no question that he was billed incorrectly. [ i am a physician and know the ins and outs of this as well as anyone] but…. to blame this on the EMR is ridiculous or to condemn a physician for using an EMR is equally off kilter,
    if a patient calls me and i need to see their data and labs etc the EMR is amazingly efficient. it is emphatically not the result of OBAMA care that we have EMRs. they were created in part for laudable reasons but certainly have been distorted to emphasize collections. that can and should be changed without getting rid of the EMR which is incredibly valuable [even with its limitations] for doctors and patients alike

    • Max Power says:

      As a patient EMR is great also. I can log onto my provider’s portal and see all my test results and other information. That’s certainly a lot better than the silo approach where that information is sitting in a paper file in some office somewhere.

      The main reason for example that Israel got to be one of the first in the world to get the vaccine is because they’ve had EMRs since the 90s. As such, they were able to go to Pfizer and work out a deal by which they could slice and dice all sorts of very detailed data related to vaccine participants and their medical history which would really help Pfizer understand what is going on with their vaccine and discover trends and associations on a large set of recipients.

      EMR as a concept is an obvious technology to implement. Unfortunately, not all patient record systems are friendly to use from a provider’s perspective and/or some are poorly implemented.

    • MarkinSF says:

      Thanks for this clarification. It was jarring to get involved in the opinion piece by a professional only to run head long into the writer’s biases. With employers changing insurance coverage, etc. it seems essential to have an EMR system in place. Every physician has the ability to read through your complete medical history even if this is the first visit. And blaming Obamacare? Ludicrous.

    • Michael Gorback says:

      The article was not about EMR.

      EMR was mentioned in passing but I said “this is a complicated topic for another day”. The article was about billing.

      It wasn’t bad writing, it was bad reading.

  16. Bobber says:

    A big problem is that the US system focuses on curing acute problems. They want to take your money, say you’re cured (at least temporarily), then get you out.

    The real healthcare need for most people is not acute care, it’s lifestyle change. US healthcare is so bad because we have the toughest most chronic cases. I think I heard 50% of our population is obese.

    Instead of spending hundreds of billions a year on acute care, how about requiring doctors to instruct on lifestyle matters as part of the regimen. It would be nice if doctors gave out an overall grade based on lifestyle, then such grade was factored into health insurance rates.

    • Bobber says:

      Would be interested to hear Michael’s experience with lifestyle change recommendations, how they were received, and whether they had any impact on most patients.

    • sydneycollin says:

      Sadly, diet & lifestyle changes don’t make the medical establishment money. A healthy individual can’t be billed as readily as a sick one.

      When the current ‘standard of care’ requires doctors to treat symptoms with drugs & procedures instead of correcting the root cause, they lock in a recurring revenue stream as the patient gets sicker.

      Just look at the past few decades as obesity & the cost of health care have both skyrocketed.

    • VintageVNvet says:

      Several errors in your comment B:
      1. USA medical services delivery system focuses only on symptoms, and does not care a fig about any ”cure.”
      2. According to most recent read, 70% of USA folks are ”overweight,” and approximately 40% classified obese, both a national shame, far shore.
      3. Having been on ”free clinic” care in the very long ago past, and in preference to also free care at college infirmary, IMHO, it’s WAY past time for the really good folks in USA medicine to insist on free care for everyone, but only for clearly defined minor ”trauma” and actual ”health” care,,, along with, as you say,,, intense ”help” with re bad and ”badder” habits of all kinds…

    • Otishertz says:

      Docsters should be required to study nutrition for at least one full semester. So many illnesses can by cured or avoided with proper nutrition.

      • JoeC says:

        From a dietitian: If they just covered nutrition visits, it would be waaayy cheaper. Doctors get paid big bucks, a dietitian makes $20-40 an hour. However, the only way Medicare (which most insurances follow) will pay for preventative nutrition visits is with gastric bypass, newly diagnosed diabetes, and dialysis.

        Most care should get shifted to preventative, but in order to do that, you have to take away the high cost treatments voters (e.g. older people) love. So I am not sure what the answer is.

        • sunny129 says:

          JoeC

          There is NO money in preventive care unlike in the ‘sick care’

          Seen it, been there and got sick of it!

          Some times ‘doing nothing’ is the best medicine but there is no CPT to charge for that good advice!

      • Ed C says:

        They don’t learn anything about supplements either. No money to be made on supplements so very few doctors will suggest or recommend them.

        • Harry Houndstooth says:

          It is not possible to be in optimal health without supplements.
          Many of the Omega-3 supplements sold in this country would be banned in Europe.

          The FDA does not get involved in supplements. There is not enough money in it. Just health.

      • Cas127 says:

        “Docsters should be required to study nutrition for at least one full semester.”

        It is kind of insane that Drs. have essentially *zero* *required* training on nutritional matters (which are far more detailed and well researched than most readers probably assume…spend a couple hours plugging basic nutritional supplements into the NIH’s Pubmed and you will be amazed at the large number of simple, OTC interventions that could be at least *tried* before some $5k+ a yr Big Pharma darling).

        • Paulo says:

          No offense, but I think all doctors are quite well versed in good nutrition and lifestyle choices. Okay this is in Canada, but my old neighbour was quite heavy. On one visit her doctor told her to seriously lose weight by changing her lifestyle and eating choices or she would have to find another doctor. Granted everything about people is more complicated than it appears, especially the whys of behaviour and eating, but the point was made.

          The EMR sytem is the way to go, at least in BC as all records/results can be accessed by both physician and patient the same day as any tests or procedures are done. Furthermore, for referrals and specialists the history and records are immediate, including for future ER visits. Even the ambulance EMTs have access and act accordingly while enroute.

          Of course there are no supplemental billings or co pays for anyone. That seems criminal, but I won’t go there.

          However, one thing is the same in both Canada and US and that is the pandemic televisit (for want of a better word). If I now call the doctor’s office for an appointment I get a phone call back from the doc in lieu. If necessary, an in-person appointment will be scheduled. So for the same issue, the doc bills the medical plan for two visits. And like most men I never even call the clinic unless I have no other choice. I used to get a yearly physical….just because. This year I piggy backed a call my wife was on and asked for a lab referral for all the usual suspects a 66 year old male would/should have. I could get the lab work done at our local primary care clinic. The PSA cost $9 as I have not had prostrate issues so it was considered discretionary, but everything else was free with results online the next day.

          I have POA for my elderly tenant (he has no family) and am registered at his clinic as such. When I see health issues needing attention I email his clinic, the receptionist prints out the email for his doctor or associate, and then I either receive a follow up phone call or they schedule him an appointment which he faithfully attends none the wiser. The flexibility and personal service is fantastic. This is a positive thing to share and is the only reason why I mentioned it. It’s cheaper for the medical plan than $1k per day in hospital.

          I think being a doctor would be very very hard, especially with so many consulting Dr Google before every visit. If you find a good one be thankful. I cannot imagine the emotional toll it takes to deal with impending death or continual health problems with other people.

        • Cas127 says:

          Paulo,

          Modern nutritional studies go way, way, way beyond the food pyramid and fat (and carbs) bad (or is it the reverse?…).

          A number of very high profile pharma drug classes were first detected (and subsequently refined) in naturally occurring plants (including corticosteroids and statins).

          Run a list of vitamins and herbs through Pubmed and you will find many thousands of peer reviewed research papers supporting at least trial use of natural herbs and vitamins to attempt treatment for otherwise recalcitrant medical conditions.

          I group all that knowledge under the heading of “nutrition”…but the vast majority of Drs. are not required to take a single course in that area…which has advanced far, far beyond ye old food pyramid.

          And your average practicing clinician lacks both motivation and time to expend much energy working with Pubmed…it is just easier to cookbook their way through “consensus treatment standards” (regardless of individual effectiveness/side effects…there is always another new on-patent Pharma product being pushed by the sexy sales rep).

          I think you might be surprised at the number of “nutritional interventions” (broadly defined to include any OTC pdt) that could be attempted before the MIC’s (medical industrial complex) default cookbook really has to be invoked.

    • gametv says:

      Bobber – 100% dead on. But that is the root of the problem. Healthcare for profit, but with alot of legal restrictions to competition, so that innovators dont actually start to bring down prices.

      It is going to change very quickly. The new telemedicine will convert to AI health. We need only two changes. First, get rid of the doctor’s monopoly on prescription drugs. Two, have the FDA fund research on all nutritional, supplements and lifestyle interventions and prevention techniques. By doing that, we could have those types of things added to standard of care and drastically reduce all those dangerous pills people are taking.

      • Michael Gorback says:

        Doctors don’t control prescriptions. The government decides what requires a prescription.

        You can counsel people until you’re blue in the face but unless the receiver is on, you’re not going to make the junk food and recliners disappear.

        The FDA does need to look into supplements but not for the reasons given above. The FDA has oversight but rarely does anything in terms of claims or quality.

        Arguably the supplements are as dangerous as the pills.

        Back when St John’s Wort was all the rage someone tested various brands. The percentage of the active ingredient was zero to over 20%. Just switching brands would change your dose. But it was “natural”.

        So is digitalis, and you can make your own by growing purple foxglove and boiling the leaves. Digitalis has a very narrow therapeutic range. One batch might not work and the next batch could kill you. Or you can buy digoxin, which is of known dose and purity. But it’s not natural.

        Coronavirus, scorpion venom, and poison mushrooms are natural, gluten-free, non-GMO and “organic”.

        Right now you can sell anything as long as you don’t claim to treat a disease. I can bag up my grass clippings, add some flavor or coloring in a blender and market Dr Gorback’s Pain Reliever. Pain isn’t a disease. If I advertise it to treat arthritis I’m in trouble. Dr Gorback’s Pain Reliever would be totally organic. No chemicals or pesticides. Just grass and some dog feces and urine.

        Most bottled spring water doesn’t come from springs, and springs can be loaded with bacteria, parasites, and chemicals from groundwater runoff.

        If I had my hands on the reins of the FTC (not the FDA) the minute I saw a TV commercial with those stupid jellyfish accompanied by a claim it “enhances memory” (you can’t say it treats Alzheimers) I’d be on the phone the next day telling them I want proof of that claim on my desk in 2 days. How much does a jellyfish remember anyway?

        Education in med school about diet? Which one? Keto, South Beach, Mediterranean? Spin the wheel. Besides, today’s knowledge is tomorrow’s fallacy. Sorry about the recommendation for those nasty egg white omelets. We take it back. Now eggs are good for you.

  17. Danno says:

    In the words of Steve Earle:

    “Look around
    There’s doctors down on Wall Street
    Sharpenin’ their scalpels and tryin’ to cut a deal
    Meanwhile, back at the hospital
    We got accountants playin’ God and countin’ out the pills
    Yeah, I know, that sucks Ã??Ã?? that your HMO
    Ain’t doin’ what you thought it would do
    But everybody’s gotta die sometime and we can’t save everybody
    It’s the best that we can do

    Four score and a hundred and fifty years ago
    Our forefathers made us equal as long as we can pay
    Yeah, well maybe that wasn’t exactly what they was thinkin’
    Version six-point-oh of the American way
    But hey we can just build a great wall around the country club
    To keep the riff-raff out until the slump is through
    Yeah, I realize that ain’t exactly democratic, but it’s either them or us and
    And it’s the best we can do.”

    • Sam says:

      Steve Earle addendum – “The Devil’s Right Hand”

    • 728huey says:

      Or as “Weird Al” Yankovic once parodied Madonna in “Like a Surgeon “:

      It’s a fact, I’m a quack
      The disgrace of the AMA
      Because my patients die
      Yes my patients die
      Before they can pay

  18. 2banana says:

    Wonder how much better and more affordable US Health Care would be if they had to abide by the same monopoly laws that the average motor garage has to abide by (or enforced to).

    1. Publicly post prices so customers can comparison shop
    2. Give out estimates that they must honor
    3. Competition all over the place
    4. Collusion is severely punished
    5. One price is charged for the same work no matter who is the customer and no matter what kind of insurance they have

    • sydneycollin says:

      This is why their lobbyists are paid handsomely.

    • Michael Gorback says:

      And there are no motor garages that do fake oil changes, charge you for brake jobs and show you some used brake garbage that they allegedly replaced, etc.

  19. Otishertz says:

    The most striking thing about the pandemic is that no one anywhere is clamoring for universal health care or free basic medical services. Mandatory injections, yes, but no changes to the way the medical/insurance cartels do busines. Just crickets.

    The medical services industry is beyond disgusting and corrupt. Meanwhile they all present themselves as heroes with halos.
    Most bankruptcies are still due to unpayable medical bills.

    The average serious medical proceedure costs multiples of the average life savings. Its price should reflect local incomes, not the rapacious greed of the participants in this evil fraudulent system we pretent to call Health “Care”.

    • gametv says:

      I went to an ENT doctor the other day and was informed that they might do a procedure that is not covered by my insurance. Basically, it is a little tube with a camera that looks in your nose. $1500 for that. No thank you!

      Every patient that steps in that door they try to stick that up their nose and pull out $1500.

      Medical billing is pure FRAUD!

      I will be quite happy to take my business to a good AI system in the very near future, and tell those overpaid doctors where to go.

      • sunny129 says:

        All ‘proceduralists of any kind’ ( ENT, opthamology, ortho, surgeons, pulmonologist, Gastro enterologists ++) make more bucks by doing that ‘procedure’ ( whether medically indicated or not!) using, needles, catheters. tubes and scopes of all kind. There are more CPT codes for all those ‘lucrative’ procedures!

        For invasive procedure of any kind, I pause!

  20. Patrick David Lewis says:

    the Manx NHS in the Isle Of Man is great for care, however since the Doctors have gone modern they spend the time typing rather than dealing with the issue, also they say only 2 issues per visit so you can get fluxed i had a blood test at 9ish in the morning and had my ear was cleared out by the same nurse in the same Doctors surgery but had to come back 3 hours later to get it done

  21. RH says:

    Doctor at the risk of getting you censored, is something funny not going on as to certain, low-cost medicines that some studies have suggested actually work as to this virus? What will happen when the Lambda and C.1.2 variants eventually share genes, which seems inevitable given their spread? Will prior infection give us some protection? If not, vaccines will become less effective. Will that not be catastrophic to the economy?

    • Nick Kelly says:

      At least if you take the horse de-wormer you probably won’t have worms.

    • Paddy Jim Baggot MD says:

      academic medicine, the CDC, the FDA, medical journals
      have been eposed as more corrupt the gangsters
      even more corrupt than insurance companies (ok, just kidding on that one)

  22. David Hall says:

    There are patient advocates who work for a fee and/or commission on the money they save negotiating bills with providers and insurance companies. I read about incidents of double billing, billing for unnecessary procedures, unnecessary tests, lack of pricing transparency etc.

    I looked into it after recent emergency surgery. So far the bills seem reasonable. The anesthesiologist wants $2000. The hospital wants $21,000. I have not seen all the bills yet. Health insurance pays part of the bills, I pay the other part. My health insurer denied payment of a few bills. I presume those providers will want me to pay the bills my insurance denied. My hospital is in network.

    I am using home care workers to do my chores as I rehab at home instead of in the hospital. Today my physical therapy supervisor said she will discharge me from PT in two weeks.

  23. KakashiHatake says:

    Serious question: why not ignore collections? The common refrain is it’ll trash your credit. But if you’re not doing it constantly, why not just not pay, especially if you otherwise have good credit and assets? I’ve been doing that with medical bills that I think are inflated and it hasn’t hurt me yet, ableit my sample size is less than a handful of instances. Curious if anyone else has tried this or knows the limits of such recourse.

  24. CRV says:

    About EMR: most big IT-projects involving digitizing paper trails, have the goal to cut out the secretary and is mainly implemented for the managers, the overseers, to watch whatever the workers are doing and having a track record of what went wrong by who. It’s rarely to make the workers life easier or more productive. IT-projects have the nasty habit to become too extended for the purpose it was intended for. Not to mention all the open ends and bad security issues. Pen and paper beats IT in most situations where manual labour is involved. [a confession of a former IT specialist].

    • MonkeyBusiness says:

      I work in software development and yes IT is overrated. US tech companies could not even come up with a way to help the nation deal with Covid. It’s all BS.

    • sunny129 says:

      EMR or HMR or whatever was started as efficiency in patient billing and later this, morphed into EMR/HMR but the focus remained the fees to be collected. Patient and along with Dr was left out from any meaningful input! It was more suited for ‘bottom line; focus of MBA than the MD!
      So, what did you expect?

  25. Joel c Johnson says:

    After working as an orderly to put myself through college I am afraid to get sick. Doctors bury their mistakes and most dispense meds on the basis of symptoms not tests! No money in cure money is in treatment!

  26. Dr. J says:

    Amen. I became an eye doctor (new term, Optometric Physician) in ’84, influenced by my 1957 EENT, Dr. Isadore Fine. Dr. Fine did his magic, I got a pair of glasses, and I could see. EENT’s are long gone, but I wanted to be the country doctor and did so. 4 years of Military medicine in a hospital was a great primer to private practice. Opened cold in ’89 and the change from then to now makes me glad I hung up my ophthalmoscope. Paper charts were fine. Photocopier and fax machine were fine, but EMR was a nightmare. The interoperability of pushing a button to transfer records was and is a joke. Anything you transmit generally requires a phone consult, and try calling your corporate doctor…go ahead…try. Fax your questions or email it, but God forbid you want to talk to him or her. Eventually doctors set up “here’s my cell” to get around the corporate rules, and calls after-hours are normal. Your doctor DOES care. But we bought into the Obamacare snooker to the detriment of ALL concerned. Except Obama. Everybody wants to pay less (businesses, patients, insurers) while everyone has their hand out for a piece of the action (billing services, benefits coordinators, sequestration (thanks Barry for that one), and endless paperwork and bureaucracy). I’m sure Dr Mike could write a book, but I’m blessed that my primary doc is a personal friend and we both were solo flyers, so we both “get it”. Dr. Mike…get a new PCP buddy and concierge each other. And do send it to the Medicare IG. Occasionally, they love stuff like this…but doubtful it’ll change. I miss ICD-9 and CPT without every flipping’ modifier. Yeah, that makes it easy and time spent in a code book, is less time with a patient. Thanks for letting me vent as well.

    • Michael Gorback says:

      Don’t forget if the occasion arises there’s an ICD-10 code for being sucked into a jet engine

  27. Twinkytwonk says:

    As an Englishman who has a wife with Crohn’s disease I was offered a job at Florida uni in 2016. The costs for health coverage for my wife was around 6k a month and this was the excess we would have to pay. I turned down the job because of this.

    For the majority of Americans it must be terrifying

    • Ed C says:

      Yes it is. We count the days until our 65th birthday. That’s when we reach Medicare age and have a chance to avoid medical cost bankruptcy.

      • Michael Gorback says:

        Medicare is the best insurance you can get in the US. I predicted that 20 years ago and people thought I was crazy.

        Well, my prediction has come true but people haven’t changed their assessment of my mental status.

        This isn’t necessarily bad. When settling disputes I just look at the other person and say You do realize that I’m a lot crazier than you?

        • lenert says:

          The Medicare Administrator is paid like $250k a year – the CEO of just one private health insurer makes like $25M a year.

        • lenert says:

          I was looking forward to medicare but they really motivate you into an advantage plan and it’s fear – fear of getting stuck with a bill while you’re waiting months for a medicare employee to process your claim instead of going to the private, subsidized, insurer and getting it done on time – it’s just outsourcing the claims processing. so now, basically paying more than standard medicare to a subsidiary of the same insurance company that covered us in the state exchange through a different subsidiary and the same CEOs gets his cut – it’s all a racket we swallowed because we’ve been told the government can’t do anything but for the last 30 years most everything in government’s been privatized but when shtf it’s never the fault of big bidness.

      • Rcohn says:

        Monthly Medicare premiums run $148.50/ month / person and is deducted from social security. Annual deductible is a further 201$/ month .
        Supplemental insurance / person can run another 150$-300$/ month and covers the vast majority of procedures
        Total insurance costs even under Medicare insurance costs can run 400$/ person / monthly .

        • Rcohn says:

          Sorry Deductible is 201$/ year , not annually

        • Wolf Richter says:

          Not if you have a Kaiser Advantage plan that covers everything all the Medicare “Parts” cover, plus stuff that Medicare doesn’t cover, plus it has maximum deductibles and max out of pockets. This is the case in California. Prices vary depending on what city you live in, from $0 to $84 a month, plus the amount for Medicare Part B, and that’s it.

          The whole Medicare situation is very complicated. And people need to get informed on their own. But it’s worth it.

        • NARmageddon says:

          Wolf mentioned Medicare Advantage plans somewhere above. I think that would be a useful topic on its own.

          For starters, is not Medicare Advantage simply a scheme to insert an insurance company between the patient and the actual Medicare system? I mean, the patient gets certain freebies and price reductions on certain things, but there must be a downside to the patient, or else the insurance company would not do it. And think it is not just the monthly premiums, since they can be relatively low compared to typical insurance. For example, there may be some real limitations on care relative to being a “Medicare direct” patient, whereas US Govt pays some fixed amount to the insurance+providers. Sort of like a subscription model?

          I know this is a big topic, and I have researched it a bit on the web, but very interested in getting some good links and/or opinions from knowledgeable people here. It looks to me like insurance companies are advertising Medicare Advantage plans very heavily on TV, and tout all kinds of benefits, but are not real keen on people understanding what MA plans *really* mean.

        • Wolf Richter says:

          NARmageddon,

          As with all insurance plans, whether or not Medicare Advantage is good for you depends on the insurance company. I was talking about Kaiser plans. I would be very leery of Anthem plans. Anthem is the worst, in my experience.

          Providers of Advantage Plans get paid by Medicare to take care of their patients. That’s the principle. Whether or not they do is another story. For example, Kaiser’s plan includes drug benefits that gets rid of the Part D “donut hole” that so many people with Part D complain about. There are a lot of other benefits of the Kaiser plan, including caps on deductibles. But you cannot get Kaiser in every state.

          All this is very complex, and based in the state, and individual needs. Medicare requires a huge amount of research. All this far exceeds what one article can do.

        • lenert says:

          NARmageddon,

          Advantage plans were born because private insurers kept whining to congress that they weren’t able to make enough money off old people in the private market so the government started by outsourcing the medicare claims processing to the privateers. The complexity is necessary to obscure things that should very simple but at it’s core the insurance business makes money by collecting premiums and denying claims. Read Wendell Potter, former Aetna exec but do it on an empty stomach.

    • tfourier says:

      @Twinkytwonk

      Having dealt with the US system for decades and very familiar with often offhand way the NHS treats people with long term chronic conditions the problem was you believed all the horror stories, basically propaganda, you saw in the UK media. Just rehashing political partisan US media stories.

      My partner has MS and Crohn’s and her kid had a heart murmur (they grew out of it) and I have long term chronic conditions but because I knew how the US system works it never once crossed my mind to go the route you attempted. Self pay, plus catastrophic plus very active and proactive health management made for a very cost effective health care costs and immediate treatment, when needed, health care. No waiting in A&E for hours like in the NHS.

      Because of my family situation I get to not only compare but mix and match systems. For general and specialist health care the US is second to none. But just like when hiring lawyers a little bit of research saves a lot of money. But for dentistry and optometrists private treatment in various European countries when visiting family saves large amounts of money compared with US. Dentistry is a very bad deal in the US. For certain medical procedures France is a very good deal at the moment compared with the US. If you have a native language speaker to hand.

      But the NHS. Almost bottom of the my list when it comes to medical treatment. The only system worse in my experience being the Italian system. It killed my baby nephew. In the slap dash way so typical of socialized systems. He would have lived if treated across the border in France or Switzerland.

      In my experience all the horror stories about the US medical system involve money. Which can be fixed. In my experience all the horror stories about other countries medical systems involved people dying. Which cannot.

      Thats the big difference.

    • Ron says:

      Healthcare in England is even more scary. Six month wait for emergency care. One week wait for an ambulance. British healthcare is great, until you need to use it.

  28. El Katz says:

    U.S system is not “health care”, it’s “sick care”. One only look at the Covid vaccine big pharma bonanza for proof. There is no benefit for the medical business to make you well. That’s your job. They just fill you up with useless medications to mask symptoms that could likely be corrected through a few changes in lifestyle (like putting down the bag of cheezy poofs).

    The doctor that I was seeing in CA went to a concierge practice to avoid being forced into what he called “corporate medicine”. He said he entered medicine to spend time with patients and not computers. It was a great practice and the last time I would say I had a relationship with my medical practitioner… appointments at any time, usually shortly after you called. Had his cell phone number (which he answered). Would make house calls if need be. Would see out of town visitors to our home if they fell ill, rather than force them to a “doc in a box” or ER. AFAIK, he didn’t move to the electronic medical record scheme due to the expense.

  29. gametv says:

    There are really two problems in medicine. First, doctors have been brainwashed to believe that prescription medicines are the best medicine. Patentable drugs are the only ones that can afford double-blind studies that meet the stringent requirements to be included in standard of care, so doctors completely ignore nutrition, lifestyle, supplements as methods of treatment. But most doctors are CLUELESS about anything that is not a prescription.

    Second, the use of a doctor to ask virtually the same questions of a patient and then deliver the same exact recommendation as hundreds of thousands of other doctors is horribly inefficient. And it doesnt even deliver the same quality of diagnosis/treatment that could be achieved with a good AI system. A good AI system that “treats” millions of people could also run analysis on treatment effectiveness and create a “learning loop” or it could vary the diagnostic questions and could also cross reference the whole medical history of a person to get to much better answers.

    The future is to have doctors focus on procedures that cannot be done by computers and let computers do the diagnosis. Doctors can continue to treat the 5% of cases that are really difficult. And they can do more research to invent new treatments for chronic conditions.

    The only block to this future is the corrupting influence of money. Doctors will fight back and everyone in the drug business will not want to allow treatments that actually solve chronic conditions cheaply, instead of chaining patients to long term prescriptions that dont cure, but merely disguise symptoms.

    And if we get rid of the monopoly that doctors have on prescribing drugs, we can automate all of those follow-up visits that waste time and money. Just answer some questions to the computer and have it track your progress and it can deliver a better experience and results than a human every time. Because computers are great at creating repeatable systems that can be monitored and improved over time. Humans, not so good at that.

    Many doctors got trained a long time ago and their only real continuing education are seminars and literature from pharma companies. You think that guy can treat you holistically? You think he knows the latest research?

    While I appreciate the article, it focuses on minutia in billing. There is a simple solution that is happening right now. AI diagnosis. It is coming and in 5 years from now, we are in a completely different healthcare world. Good investing opportunities in this area as well.

    • Wolf Richter says:

      Gametv,

      AI the solution??? Oh boy, we can’t even design an AI system that does the basic function of driving a car safely. Every 16-year-old can learn how to drive a car. And we cannot build an AI system that can do what a 16-year-old can do. So you want me to go to an AI doctor that cannot even do what a 16-year old can do? Hahahahaha…

      Your suggestion might make sense many years from now, but today, it’s totally nuts. You go get treated by your AI doctor, I have no problem with that. I’ll write the story of how the AI doctor killed you.

      • Petunia says:

        AI can only do what the programmer tells it to do. Once you start to compare many symptoms, along with demographic and geographical data, the number of combinations can increase to huge numbers. The last thing you want is the computer giving you it’s best guess as to what’s wrong with you.

  30. LK says:

    Hellcare.

    *Nice.* Bravo.

  31. Brisket says:

    Watching the US health system from afar is like watching someone step on a rake repeatedly. And when it’s suggested that maybe the rake could be moved you’re a GODDAMN SOCIALIST GET OUTTA HERE.

  32. Raging Texan says:

    Dr. Gorback, Here’s my questions:

    Where in the U.S. Constitution (not the fantasy literature called Supreme Court OPINIONS) is the Fedgov authorized to regulate local medical care?

    here in the U.S. Constitution (not the fantasy literature called Supreme Court OPINIONS) is the Fedgov authorized to pay for medical care?

    Assuming the Constitution even allowed the Fedgov to be involved in healthcare, What is the actual text of HIPPA passed by Congress? (Hint it’s just a stupid exec program masquerading as if it were passed by Congress, which even if it did, still completely unconstitutional)

    Is it unconstitutional federal, or is it state level regulations that require me to get a script from a physician for hundreds of non-harmful drugs that have been around for decades, instead of just walking up to the pharmacy and buying what I want? (Hint just go to Mexico or dozens of other countries to buy what you need with no hassle)

    For years, the physicians lobby did a great job of getting the Fedgov to pass all kinds of unconstitutional laws and regulations that benefited them. Now that the Fedgov is eating the physicians along with everyone else, are there any physicians willing to stand up for the rights of all citizens? Or do physicians just want the Fedgov to go back to what they were doing a few years ago, stealing from everyone else on behalf of the physicians?

    Also a tip for everyone else who like me, realized the last thing a person wants is “healthcare” within the borders of the USA, here’s my tips:
    – international travel. a 19 foot sailboat can actually go around the world just fine
    – self care , everything from routine amputations to brain surgery is now taught on youtube, can be done at home
    – medical directives- write on there that you won’t go into any hospital unless the physicians receive the same treatment and bills you do
    – fight all bills – if stuck with any bills, offer no more than 5-10 cents on the dollar and let them go to collections if necessary.

    • Top-GUN says:

      Raging Texan,,,, AMEN to that brother… There is absolutely no authorization in the Constitution for federal government to be involved in healthcare, or be in the insurance business (medicare) or provide free Healthcare (medicaid) or force insurance companies to cover pre existing conditions. ….
      And yes FAT people, OBESE people, and I bet 80% of you reading this are Over Weight, need to lose weight… being fat is not good in any way…

    • Michael Gorback says:

      Texas transplant and libertarian here. We need to get rid of the 6 steps from Kevin Bacon game, where in a few steps you can connect anything to the elastic clause (Article 1, Section 8 of the Bill of Suggestions).

      The elastic clause was used by Hamilton to create the First Bank of the United States, predecessor of the Fed.

      We need teeth in the 10th Amendment.

      This is above my pay grade but so is most of Wolfstreet.

      • Raging Texan says:

        Dr Gorback, you may also be interested to read the TX bill of rights, and compare the actions of state officials to said document.

  33. John says:

    Hey Mike,
    Thanks for sharing! I’m sure when it’s noticed about the billing, “oh we made a mistake,” I’m sure they know as in, Greed!, as you put it. A while back I was hearing about or reading about Medicare being able to negotiate drug prices. I’m sure my Medicare from social is going up.

    • Michael Gorback says:

      Funny thing about that. When they enacted the legislation for Part D (medication coverage for Medicare) they put in a clause that prohibited Medicare from negotiating drug prices.

      It’s the largest insurer in the country and it has NO market power.

      Gosh I wonder how that happened? I call Part D The Uniform Gift to Pharma Act.

      • sunny129 says:

        MG

        VA is allowed to negotiate for prices with Big pharma but Medicare was specifically forbidden by law, enacted under Dubya as a gift to Big Pharma for all their contribution to him. There is even a PBS documentary (front line?) even listing them by item. It was day time robbery but no outrage from any one! Wonder Why?

  34. Anthony A. says:

    Mike, nice write up! Thanks for the effort!

    After 13 years on Medicare with a supplement (wife and I), I have lots of stories about screw ups, over charging, fake billing, doctor’s offices not coding things right, and on and on. But I have mastered the use of the system. All I can say is “be prepared”, do your diligence ahead of time, and question EVERYTHING!

    I have also fired two doctors over blatant Medicare billing mistakes and their unwillingness to help fix the problems they created.

    • #42 says:

      imagine 100 years ago and how your complaints would echo …

      • #42 says:

        New hip, new shoulder, new knee, quit f n whining

      • Anthony A. says:

        I wouldn’t have any complaints 100 years ago as there was no insurance and doctors made house calls. The only paperwork would be a written check for the services.

        Your comment is meaningless.

  35. Gilbert says:

    Informative post for sure. Where we live one’s PCP is more than likely a PA. Otherwise a NP will be the extent of it. We have found that the older more experienced PCPs sort through the BS and get to the meat of things. Younger less experienced providers tend to order more tests than required. Some say it is to CYA and others say it is a money thing. Who’s to know?

  36. V Burckard says:

    I left the US when Obama was elected. 100% of my medical care has been in Mexico since 2009. The level of care is nothing short of remarkable, and easily paid out of pocket.
    I have had several doctors give me their cell numbers, and or email, especially when the service was near the weekend. Fortunately, cataract surgery is the only procedure I have ever needed, but I have had experiences with Cardiologists, Internal Docs, and Urologists. I’m an old man, what can I say.
    I’ll also say that, 1 test here cost me $349, specialist, hospital, etc., but was $3500, 5 years earlier in Cincinnati!!
    One last note, doctors here will never give prescriptions for months of, say,….oxy. Here, they might give you 5 days worth, then reevaluate.
    Sadly, Medicare will not pay for anything,………too much money to be saved, I suppose.
    Medicine is another issue. In ’09, I was paying $120/mth, for blood pressure meds. Now, I pay about $12. Same story across the board. PLUS, I am buying Invermectin for $1 a tab, no prescription required!!!! Just in case.

  37. Michael Gorback says:

    One more observation about drug prices.

    I travel to Italy a lot (or used to). One time I forgot my BP pills. I went into a pharmacy hoping to cadge a week’s worth of meds.

    The pharmacist didn’t speak English so I had to inflict my Italian on him. I managed to explain the situation.

    He asked me if I had a prescription. The question I was dreading. I said no, I was just hoping to get my medication.

    He looked at me with a grave expression and told me without a prescription I would have to pay for it. Also I couldn’t get a week’s supply, only 28 days.

    Ok, I braced myself for the gouge. The reason it had to be 28 days was because the pills came in a cardboard box of 28.

    The cost was 1/5 of what it is here.

    • stan65 says:

      Don’t you just love Italy. I once turned up at the airport in Sicily, rushed out of the plane to be first in line at Avis counter, passport, cc, license, all going well, holiday is starting. The guy at the counter says, sir, your license expired 5 years ago.

      That drowning feeling.

      Guys says, go behind these counters and see the other guys. There it was Don Vito Cash Cars.

      Can I have a car prego? Certo signore, just give us cash. No id or passport or license required. Only in Italy.

      Well, probably Greece and so on, but after this episode I made sure I check my docs yearly.

      • Michael Gorback says:

        Italy is so practical and laid back. In 2008 my daughter tripped an alarm going through the metal detector. The security guard waved her through and said it was probably just her shoes.

        Check out the requirements to enter the UK vs Italy. The UK has insane requirements, whereas Italy is like “OK, you got past the airport. We’re cool”.

        Nel mio cuore sono Italiano.

  38. Taint Boil says:

    Just got out of the hospital for bad infection that normal antibiotics couldn’t kick … one day $40,000.00 … and I’m not kidding. If I pay in 7 days I get a 15% discount (my share is $3100).

    • Petunia says:

      How is that not straight out fraud?

      • Paddy Jim Baggot MD says:

        its clearly fraud. but dont only look at the total bill.
        people go to a hospital for medical care-by doctors.
        they dont go to a hospital so they can have a room that costs 50 times a hotel room.
        what they really came for is exam by doctors, medications
        sometimes surgery.
        the cost of laying on a hospital OR table
        may be twenty times the cost of having the surgery

        its like i went to the repair shop
        it was three hundred to fix the transmission
        but it was $10,000 to use the garage.

    • Paddy Jim Baggot MD says:

      prices have become absurd

  39. DR DOOM says:

    HCA bought Memorial Mission hospital in Western North Carolina for $1 billion. The billion went into a “trust” . Good night Irene on that billion. My wife had a hip replacement . No one to help her get to the can . The nurse did not know which side of bed to get her up on which is important after a hip. No one to adjust the stool for the can to prevent a dangling limb with a new hip. I watched every thing they gave her, read all the labels. At times I was the nurse, good grief. My wife delivered babies at that hospital for 30 years. She was pissed at the new management. It’s all about the corporate needs. As Edward R Murrow said . ” Good night and good luck”. We do not fault the nurse'(s) . The Ortho Floor was miss-firing at the top.

  40. sunny129 says:

    This health care’nightmare’ in America won’t change until there is a collective and strong message from the voting public Those getting healthcare coverage via their employer want the status quo and dread single source insurance. But many people don’t realize that we have already TWO sources of single payer system. One is called MEDICARE and the other VA system.

    There are lawmakers who favor vested interests over public interests. This not a secret! All voting in Congress is a public record! Be mindful whom you are send to DC or State capitals! it is the choice for the citizens if one desires andv demands change.

    • crazytown says:

      As someone with a chronic illness, I would love the employer health insurance scheme to implode. The fear of losing a job and not being able to pay for life saving medication, having to carefully read the plan for any prospective new employer, clenching your butt each year at open enrollment seeing what changes they made to the plan for next year that will result in higher cost or forced changes in the prescription formulary.

      I think it’d insane that health care is tied to your employer. Idiotic system.

      • crazytown says:

        Also from my experience navigating the system, I am skeptical about everything relating to Covod treatment as nobody has cared anything about health before now (they claim to care about health, but all I can see is prescription drug pushing), so why would I suddenly trust the mega corps, drug companies, and billionaire weirdos who have opinions about everything and get worshipped in this country.

    • Petunia says:

      For many years during the GFC and before I had good health insurance, but couldn’t afford to use it. The deductible was $1500 and I was lucky to have $15 extra a week. A simple $20 co-pay was enough to insure the insurance was useless to my family. We need a single payer universal healthcare system.

  41. Masked Ghost says:

    In the USA if the treatments or the disease doesn’t kill you, the bill gets a 3rd chance to finish you off.

    Whiskey tango foxtrot

  42. Johnny Ro says:

    USA healthcare is a desperate stinky for-profit abomination. There I said it.

    Individuals working therein are good people, many of them.

    For example of how it might have been, check out Atul’s story. He works on the USA too, from within and without. No higher authority on US healthcare.

    I hope the link is OK with Wolf and his site

    https://www.newyorker.com/magazine/2021/08/30/costa-ricans-live-longer-than-us-whats-the-secret

  43. Mark says:

    Doctor Gorbek, I hope you can write something about how to find a decent doctor and hospital in these dreadful times. Few of them will give covid patients anything for early treatment. And once hospitalized they race to get you on a vent, which is the kiss of death.

    We have a few caring doctors like the FLCCC publishers. But how to find them?

    • Cold in the Midwest says:

      Not sure on finding a quality hospital but did find one way to help on the primary care side. I signed up for “direct primary care” which has worked out well.

      I pay a monthly fee to the Doctor’s office which is constant regardless of how many appointments I do/don’t have. The monthly fee is out of pocket – no involvement of the insurance carriers (his office does not bill or involve insurance companies in any way.) Currently $65 per month.

      The entire experience of seeing him is different. I can request how much appointment time I need and he is relaxed and calm during our time together. His office is also quiet if I need to wait – I’m typically the only person waiting. And I’ve never had to wait more than 15 minutes.

    • Michael Gorback says:

      Unless you’re in the system (doctor, nurse, etc) it’s almost impossible to figure it out. Your best chance is to start out with a good one by luck of the draw. Doctors are like birds. They flock together. Good docs don’t refer to bad ones. I had a list of doctors from whom I wouldn’t accept referrals. Sometimes it took years but then I’d see they were disciplined by the medical board.

      Speaking of the board, you can look up disciplinary actions. It’s public info.

      Pharmacists, physical therapists and others often have a good handle on things as well. They don’t see the docs in action but they do see how they make decisions and handle problems.

      Certain drug and equipment reps can help. When doctors are doing implants like knee replacements or spinal cord stimulators there are usually equipment reps in the OR. Those folks see a lot of surgeons in action. However, they don’t see them in the office or at the bedside.

      Don’t use online review sites. I’ve seen docs with rave reviews who I know were disciplined by the medical board. You can hire a service that will help polish your reputation.

      They give you an iPad with special software on it. After a visit you give the patient the iPad and the patient answers a bunch of questions. The data are then uploaded to the company, which then distributes the “reviews” to online sites.

      Here’s the trick: these sites usually won’t remove a review. You only give the iPad to your happy patients. Given enough volume the bad reviews get scrolled down the page and eventually shoved onto subsequent pages that aren’t read much.

      It’s a perverse inversion of Gresham’s Law: good reviews chase out bad.

      There was a time I’d try to respond to the reviews but you’re hamstrung if the reviewer uses their real name. Under HIPAA you can’t even acknowledge that you treated the person.

      It didn’t matter. All my business was word of mouth or referrals. I only advertised my cash program so people would know it was there.

      My rule of thumb for pain specialists is that on a 5 star system you should have 3 stars – 1 star reviews from the drug seekers you kicked out of the practice claiming you have a bad bedside manner and 5 stars from the people you helped feel better. 4.5 stars on average is probably a candy man handing out drugs or someone paying $500/month for the iPad deal.

  44. bill says:

    Is there a good place to look up Medicare rates?

    • Michael Gorback says:

      They are published online.

      Medicare is divided into regions that are administered by private companies. They each have their own web site. For instance, Texas Medicare is run by Novitas. If you go to the Novitas site you can enter the info and see the price.

      However, it’s not a simple system. You have to know the CPT code and understand how to interpret price differences between doctors who accept Medicare assignment and those who don’t. Those who don’t can charge more but Medicare takes forever to pay.

      You’ll also see charges for office vs hospital services. Usually you have to enter your region because Medicare reimburses based on their determination of how much it costs to run a practice. So large metro fees are a bit higher than the boonies.

      See how easy it is? /sarc

  45. Chauncey Gardiner says:

    Primary care physician told me the other day that physicians and nurses are leaving their practices in droves. Besides the covid-related effects on practitioners, including PTSD from stress and patient mortalities, this is the only way they can have a voice in a system that’s now managed by MBAs with no experience in the practice of medicine who are driven solely by money and profits. I mean, what did those in positions of political influence expect would occur when clinics are being bought up by private equity firms and large hospital chains, EMR systems are based on billing reimbursement from large for-profit insurance companies rather than patient care, and there’s a steadily increasing concentration of medical care in ever fewer hands. As former Berkshire-Hathaway Vice Chairman Charlie Munger once said, “Show me the incentives, I’ll show you the outcome.”

    This isn’t rocket science, but it does demand a political solution free of the politicians’ own monetary incentives that effectively prevent them from making decisions based on the greater good, under an argument that monetary payments to politicians comprise constitutionally protected “freedom of speech” per the Supreme Court’s Citizens United case decision. The quality of our healthcare system compared to the costs and outcomes in other Western countries has become an embarrassment, and it’s clearly not the fault of those practicing medicine.

    • Michael Gorback says:

      The growth in hospital administrators is vastly higher than the growth of physician supply, similar to colleges and universities.

  46. Depth Charge says:

    At this point in time, for most middle class people, the best plan of attack is to start sheltering wealth in the way of gold and other stores of value and have next to nothing in the bank or anything obvious of value should the United States of Greed run you over. Then, when you invariably have to file bankruptcy because of your $700,000 hospital bill for a week’s care, you aren’t really destitute. Fock ’em.

  47. Concerned American says:

    Time to eliminate health care insurance other than for hospitalization. Also time for the government to step in and increase the number of medical school places available for prospective doctors. Let consumers have a wider choice of doctors and let them negotiate their individual medical charges. As long as their is medical insurance patients have no great incentive to negotiate lower prices for their medical care.

  48. drifterprof says:

    There are always tiers of health and medical care, where access depends on how much money one has. In the United States, the lowest “safety net” tier was always confusing to me. It seemed like the emergency room “safety net” system was an extremely expensive and inefficient system for handling low-tier medical care.

    I’ve lived in other countries where the lowest tier was very clear. When I worked in Northern (Turkish) Cyprus, if you had middle-class income, you avoided the waits involved in the lower tier. The middle tier was very reasonable. Around 2005, I paid $1,200 for a hernia operation including two nights in a small private hospital.

    Here in Thailand, which like the United States is among the top 20 or so countries with the worst wealth disparity (as calculated by Gini coefficient), the lowest tier is very clear. A previous Prime Minister instituted a law that any poor person can get consultation at a hospital for about $1, and most drugs seem relatively cheap (helps out the subsistence villagers).

    Being married to a Thai, I’m covered by that system. For example, cholesterol blood tests and consultation with doctor costs me about $10 or so at the government hospital. Like any low-tier care, spending any significant time there requires going into an almost stoic trance mode. Before pandemic, I would see a couple hundred or so people sitting shoulder-to-shoulder in large waiting room. I could almost see diseases wafting around in the air.

    With private hospitals here (some geared to medical-tourism) the safety-net tier is easily avoidable for reasonable costs (often less than what one would pay even with insurance in the United States).

    It occurs to me that a major cause of no clearly defined lower tier of care in the United States may be fear of “socialism.” Also, perhaps establishing a clearly defined lower safety-net tier would tarnish the United States’ fake veneer of exceptionalism – a supposedly exemplary country chosen by God where everyone can thrive if they have the “right stuff.”

    It’s so complicated I could easily be wrong — my defense is that it’s so confusing to me. It’s like there’s some fascist oligarch planning strategy to divide and confuse.

  49. Jseldomposts says:

    For primary care, there is an alternative that works for some people. Direct Primary care, https://www.dpcare.org/ ,is a listing of providers, many of whom are concierge doctors – offering fixed annual fees for any and all primary care services. This cost me more than insurance co-pays and deductibles, but I get excellent service from a doctor who has about 300 clients, and doesn’t have to waste any time on billing for procedures. My son found a concierge nurse practitioner with prescribing privileges in his state. This won’t work for everyone, and doesn’t help with hospitals or specialists. But the health care provider and the clients just need to deal with health care.

    • Kenn says:

      I recently started using Direct Primary Care, and it really shows off the waste in the current big hospital – insurance system. All primary care covered by an independent doctor for $900 / year, blood tests and common drugs at wholesale prices with a price list posted on the web site. While my direct pay costs do not apply to my Obamacare HMO $7000 deductible, I’m still saving money and getting much better care.

      • Cold in the Midwest says:

        Likewise Kenn. One additional difference is going to see my direct primary care doctor is actually a pleasure. Seeing my primary care health insurance doctor was something I would always dread. The noisy and crowded waiting rooms, the waiting (often past your appointment time), then finally seeing the doctor who often seemed rushed and anxious.

        DPC will not solve everything but it is a good model and may have elements that could carry over to hospitals and specialists.

  50. Yort says:

    Beware of the Hellcare “facility fee” that is being used at hospitals, at least the ones in Texas over the last year or two. For example, I had one kid who broke their nose on a trampoline, and had to have it reset a few months ago. The “Facility fee” was about $9500, and the actual doctor nose reset cost was $3000, so $12,000 for about 25 minutes of “procedure”. Note that was In-Network (mostly screwed). Just imagine if it was on a vacation and Out-of-Network (completely screwed).

    Just two weeks ago I had another kid who got food poisoning, and so we took them to an urgent care facility, in network, because they needed saline IV due to dehydration. Turns out that urgent care can not do simple salt water IVs…so we take them to an in-network ER, in which they get 2 liters of saline solution via IV over 3 hours, and some med for upset stomach. The bill was $10,000 facility fee and $2000 for the doctor and IV fluids…so another $12,000 bill for 3 hours of ER visit in a 10×10 room. Pro-rate 3 hours at $10000, which would be $80,000 for 24 hours in a 10×10 room with a “high tech/expensive (sarcasm)” salt water IV drip…only in America, right…HA The irony is they wanted to do a CAT Scan for food poisoning, yet no abdominal pain (which leans towards very low appendicitis risk…which I pointed out when they used the “your kid’s appendix could burst and be fatal memorized speech). I denied as the radiation exposure is not good for young kids when the risk does not justify the procedure. That would have been another $6-$8k of fluff that would have served up higher profits, terrific hospital lawsuit liability coverage paid by the customer, and get a few years of background radiation exposure for a young kid in less than thirty seconds (machine age/tech and body fat percentage dependent, sure…no way to ensure you get the latest tech with lower radiation exposure, etc)…

    I would have to agree that HellCare for profit is kind of evil sometimes in America…but it is what we have to work with, so just try to self-educate and do your own research to become your own best healthcare advocate, for you and your family…and your wallets and purses!

    • Citizen says:

      Hearing these stories makes a man feel homicidal. The system can’t be fixed, the whole thing needs to be completely scrapped and re-done from the ground up. New, centralized systems need to be in place and probably 1/3 of administration needs to lose their jobs, tough.

      I can’t for the life of me understand why half the country does not think we should do anything about this, who is happy with the way this works, it’s truly criminal what is being done to us.

      My personal anecdote is I have insurance through a big company employment plan, broke my arm and it will still cost me my out of pocket max of $6,000 for this one and only thing that happened to me this year. I had surgery and after now I have 23 claims that I can’t figure out what is an actual bill and what isn’t and what to pay and I’m not some older guy that can’t figure things out, I’m a 40 yr old tech worker. It’s just too complicated with no way to really tell and when I add up what’s there it’s more than $7,000….plus the first $3000 I paid they say was an overpayment but that they have to send me a check for $2000 but it will take 3-4 weeks and they can’t charge back the card I used! But of course I’m supposed to pay the rest now right!?!? Homicidal.

      • joe2 says:

        Nobody needs “administrators”. And not “political experts” either.
        We need subject matter experts.

        But it’s harder to actually do something than to talk a US educated person into believing that you did something.

    • Kent says:

      Fixing the system is hard only because people will always vote back in their representatives and senators if they’re in the same party as the voter. That’s a fundamental flaw in the two party system.

      Why can’t we just change the way we bill? Pay the doctor by the hour and supplies at cost + 20%. Hospital charges can be $x/hour for an ER bed, $x/hour for ICU bed, and $x/hour for a regular recovery room. The bed and room charges should be high enough to cover all costs, including food.

      It makes it pretty easy to understand charges, and pretty easy to know if you’re being ripped off.

  51. taxpayer says:

    I have read thru all 157 (thus far) comments and nobody has mentioned “lodge practice.” It was common in the US a hundred years ago for fraternal societies and other community organizations to contract with independent physicians to serve their members. Apparently it worked well enough that “organized medicine” got it abolished. A few articles have been posted.

  52. joe2 says:

    I don’t have to read the article. Sorry Wolf my blood pressure probably could not stand it.

    I heard all about it from my doctors. From one how he and his partners were forced ( I did not ask him how – licensing, hospital rights, insurance cost, certification, office space? ) under a hospital cooperative deal. He now sees twice as many patients per day in the new Taj Mahal complex the hospital bought. It was a former huge oil company headquarters from the oil heyday.

    He’s a really good guy, all his patients really like him, and he is not a corporate stooge pushing drugs and services as some doctors have become, and I hope his patient rooms are not bugged.

    Another had to join a management group. He hasn’t changed either although they pushed a concierge practice for a while.

    One other just stayed completely independent, doesn’t even take insurance except for top-of-the-line unlimited payment.

    It is hard to find a really good doctor. And you need so many specialists.

    BTW, it seems veterinarians have gone the same way. Doggie care has exploded as a money grubbing business.

  53. kitten lopez says:

    [sign off] “Mike Gorback (screw the formal MD crap. We’re all in this together).”

    xxxxxx
    so glad to read this. thanks for fighting!
    i have a huuuge smile on my face after reading this.

  54. Brian W Selby says:

    I am a behavioral healthcare provider so I see things from the trenches. I make very little money because I do not work for the monopolized health care in my community so have no bargaining power. Bargaining power *is* the issue in healthcare costs. As monopolized hospitals have way too much bargaining power with private insurance companies. I sent this letter to every representative about five years ago. No response. It is truly a “buyer beware” situation in healthcare and it *is* getting worse.

    I am writing to give feedback about the state of healthcare in United States. Here are the reasons Healthcare has become unaffordable for most as I see them:
    1) Virtual monopolies have been created. In order to gain an ability to increase revenues, large hospitals have bought up almost all independent practices (of all specialties including “family practices”). They then use this large collective bargaining pool to raise their fees with private insurance companies. Being the only (or one of two) large hospitals in the area the insurance companies are “over a barrel” and lose most of their bargaining position for consumers. Consumers lose.
    2) Monopolized hospital systems then make all referrals to other practitioners in their company. If they refer to the few independent practitioners who have survived in the community it is against company policy. This means that consumers now have no choice but to go to the monopolistic hospital system (or face the ire of their physician). What business is allowed to function as a virtual monopoly in America?
    Consumers lose.
    3) There is no price transparency in these monopolistic hospital systems. Each hospital system has a contracted rate with every insurance and knows exactly what amount you have to pay. As an independent behavioral medicine provider, I know my contracted rate and I tell my patients (they will know this when they get their Explanation of Benefits). Insurance companies do have practitioners sign a contract that says that rates cannot be discussed publicly, but this does NOT apply to the consumer in front of you. They have a right to know what they pay because it goes towards their deductible after all. What other business in America is allowed to have “secret” pricing? Consumers lose.
    4) The addition of #1 and #2 with “for profit” healthcare. To be transparent myself I am “for profit” in my independent practice (the “profit” goes to put food on the table for my family). My prices stay low, however, because I #1) do not have a monopoly and cannot dictate prices #2) I am transparent about my prices and allow people to decide for themselves if I am “worth the cost.” #3) I compete with all other therapists in the area which numbers in the hundreds. In a system with a monopoly and secret pricing, competitive capitalism DOES NOT WORK. You can’t shop around for prices and even if you could there are few (if any) places to shop because they have been sucked into the monopolistic system. What other business in America is allowed to not compete? Consumers lose.
    Here are some possible solutions:
    1) Break up all hospital systems into multiple non-profit (remove corporate interests) “baby” hospitals that all compete with each other. Remove all possible independent practitioners from hospital systems (e.g. family practice, and other specialty practices). Make them compete with each other and send referrals to whom they see as the best possible practitioner in the area.
    2) Make all practices put their prices on a big sign in their waiting room. Make them put it on a website so consumers can shop around.
    3) Make it impossible to only send a referral in your own system (this would already be partially accomplished by removing specialty providers from hospitals).
    4) Make practitioners be paid for customer performance through competition and ratings. This would be through ratings BY CONSUMERS and not any other metric (this is how Amazon works so well for consumers).

    Thanks for your attention to this matter.

  55. Michael Gorback says:

    Guys I can’t begin to tell you how gratifying it has been to see the discussion this has generated. This started as a rather minor report of a billing dispute and I think we’ve expanded the discussion to cover about a dozen blog articles.

    Y’all are an inspiration for me – even the critics. Especially the critics.

    A quick poll: what do you want to discuss next? Supplements and natural remedies. Comparisons of various countries’ health systems? I guarantee I can piss off a lot of people because no matter where you go in the world health care is FUBAR.

    I’m going to try to avoid covid because it’s too political. I was on the verge of writing about international travel and covid testing requirements today but I backed off because I don’t want to kick the hornets’ nest.

    However, take a look at the shit storm of class action suits over travel insurance and covid. I might write about that but I need more legal info first and an agreement from Wolf to severely restrict covid arguments.

    The problem I want to get into is that many countries require that you have travel insurance but right now the lawyers are feasting on class action suits about travel insurance. Will you be able to even get travel insurance? My credit card provides travel insurance. Will Italy accept it and is travel disruption due to covid a covered event?

    Again, thank you for your interest and contributions.

    • p coyle says:

      i cast a vote for supplements and natural remedies. and how to tell a good one from a bad one. i would be very happy to see you write more articles on all sorts of medical topics, as it’s definitely something that people need to be more informed about. thank you for taking the time and effort to write this.

    • Yes says:

      My vote…

      I would love to see some candid discussion from an insider’s point of view about pharma meds that kill or maim.

      My father-in-law died from a physician overlooking a pharma “black box warning.” Father-in-law was 80 with memory issues, trusted the doc. The drug was contraindicated because of other meds he was taking.

      I personally have permanent peripheral neuropathy from Ciprofloxacin. 12 years now. In those 12 years *Not A Single Doctor* has suspected Cipro as the cause, even as I brought up my suspicions (such as symptom onset coinciding with the course of Cipro). I’ve been tested for everything else though. Every physician I’ve seen has been completely blind to considering Cipro’s side effects. Meanwhile the research on PubMed now clearly shows the connection between neuropathy and Cipro. Yet I cannot get a doc to have this conversation with me. I keep looking for one that will.

      So my question is…what on Earth will it take to get physicians en masse (including med schools) to acknowledge the incredible side effects of pharma drugs. Why aren’t these effects taken more seriously? For God’s sake, I took a med that f*cked up my life. That’s the kind of thing that should be Very Hard To Do.

      That’s the state of our medical industry from my perspective: dangerous and incompetent.

      Really appreciated the article, and all the comments too, as usual on Wolfstreet.

    • Paddy Jim Baggot MD says:

      thank you so much for this forum
      i dont think you should try to avoid the hornets nest
      i would go for it

  56. Josh says:

    are you familiar with direct primary care / direct specialty care?

    #directcare offers unlimited visits, no copays, all procedures free, wholesale meds/labs for 95% savings, furthermore #directcare can help lower health ins premiums by 30-60% in the first year

  57. NARmageddon says:

    Michael and Wolf: I meant to post the following here below Michael’s request for topics, but it ended up in the wrong place. Trying again.

    ————————–

    Wolf mentioned Medicare Advantage plans somewhere above. I think that would be a useful topic on its own.

    For starters, is not Medicare Advantage simply a scheme to insert an insurance company between the patient and the actual Medicare system? I mean, the patient gets certain freebies and price reductions on certain things, but there must be a downside to the patient, or else the insurance company would not do it. And think it is not just the monthly premiums, since they can be relatively low compared to typical insurance. For example, there may be some real limitations on care relative to being a “Medicare direct” patient, whereas US Govt pays some fixed amount to the insurance+providers. Sort of like a subscription model?

    I know this is a big topic, and I have researched it a bit on the web, but very interested in getting some good links and/or opinions from knowledgeable people here. It looks to me like insurance companies are advertising Medicare Advantage plans very heavily on TV, and tout all kinds of benefits, but are not real keen on people understanding what MA plans *really* mean.

    • Michael Gorback says:

      Medicare Advantage lures people in with what appears to be lower costs.

      However, you’re combining Medicare rates with HMO behavior. By HMO behavior I mean things like requiring pre-authorization, charges being denied after care has been rendered etc. In essence, it adds to the financial and administrative burden of a practice.

      You personally might find authorization denied by the plan.

      One thing that I like about the way Medicare is run is that if you know the rules you can navigate it much more easily and they pay fast.

      If you have a MA plan don’t expect to find many doctors who will see you.

      And stay off the ObamaCare exchange. An exchange Cigna HMO is not like a Cigna HMO plan purchased from Cigna.

      A doctor who’s on the panel of Cigna HMO is not automatically on the panel of a an exchange Cigna HMO.
      This caused a lot of heartburn when the exchange opened. People found that they had insurance but no doctors who accepted it. Why? The exchange plans paid around Medicaid rates.

      My daughter was thrilled to get a Gold plan. She went through the exchange. Then when she got sick there was one clinic in Charlotte, NC that accepted that “Gold” plan.

      ObamaCare is probably something to add to the list.

      • Wolf Richter says:

        Michael Gorback,

        You’re in Texas. The state government tried its best to GUT Obamacare, so now Obamacare is not working in Texas because it was gutted. Same in NC. It works just fine in California, where the state tried to make it work. You get what you want to get.

        When we moved from Texas to California in 2006, and changed from our Blue Cross insurance in Texas to Blue Cross in CA (before Anthem bought it), our rates dropped in half, same type of high deductible insurance. Texas is where health insurance is encouraged to rip you off. You have no idea how you get ripped off in Texas with your health insurance, compared to a state like California.

        So now you’re on Medicare. And that’s federal and you’re out of the clutches of Texas on it. I wouldn’t do an Advantage plan in Texas either. That’s just another way of getting ripped off by the insurance environment in Texas. You have no idea how badly you’re getting screwed.

  58. Eferg says:

    Dr. Gorback and Wolf,

    Thank you for this article and the future ones you promise. I have a keen interest in the disastrous costs and billing associated with the US healthcare system. This issue needs airing.

    Healthcare is essentially an oligarchy of providers and “insurance” companies aided and abetted by government. Pricing has become criminal racketeering designed to herd people to the insurers and eradicate providers independent of the oligarchy.

    Many good comments and horror stories – all appreciated. A number of solutions have been offered, so I will add my two cents here: Require price transparency and that the self paying person get “most favored nation” treatment on pricing. That would significantly change the pricing dynamics.

  59. Andrew says:

    Entertaining commentary and analysis.
    Thanks to Wolf for recognising a talent.

  60. lenert says:

    Decades pass and more ink and blood spills for money. Health coverage is definitely something that’s been lacking on Wolfstreet. i’m no expert but it seems like there’s still a lot of truth in Kenneth Arrow’s 1963 essay on “Uncertainty and Welfare Economics of Medical Care.” And with respect, MDs have an excellent, if not the best, union. If Docs had to compete in a world market for labor like the rest of us rates would be a lot lower. I keep reading that docs in other countries get paid less and provide better outcomes – i think that’s before the PE guys get their cut.

  61. Lawefa says:

    @Michael
    Great article to start the conversation and inform people. I’ve had extensive discussions with some really good doctors and they day alot of the same stuff you have mentioned. They have had to combine with large corporate entities to survive and stay in conformance with laws.

    The whole medical industry is a boondoggle with billings and doctors are frustrated with the system too. Usually out of a $250 visit, a doctor pulls maybe $40 to $50 per visit. It’s a sham that has been over legislated which has stacked the cards against good people. We all lose with the current setup – less patient/doctor face to face time, lower patient care, and higher (at times illegal) billings that just tax the common people. It’s broken and a long road back to correct it.

  62. Willy2 says:

    – What a nice system capitalism is right ? Squeeze the patient as much as possible.
    – The insurance companies and the government want to control the processes (and costs) as much as possible but that simply adds one or more layers of “paperwork”/bureacracy to the burden of the healthcare system.

    I have heard the 2 following stories:
    – One person (a friend) had to see the doctor while he was on vacation in Florida for a (fairly) minor medical problem (no, no details). After the doctor made the diagnosis and gave this person his medicine this person asked how much this visit would cost him. The doctor anwser was (about) $ 8000. The person then asked how much his visit would cost if he would pay the doctor without all the bureaucratic hassle. Then the price came down to (about) $ 2000. He directly paid the doctor in cash with his credit card. And I assume that even then the doctor made a handsome profit.

    – I have heard that a person without (the proper) insurance who goes to the hospital is charged (at least) 5 to 10 times the rate that is charged to a person with full coverage insurance.

    – Again, what a wonderful thing “Capitalism” is right ?? And the politicians are still wondering why more and more people are “unhappy” (to put it very friendly) with the current system called “Capitalism”. And more and more people are becoming more and more extreme when it comes to their political beliefs/convictions. And are abandoning the moderate political centre and become more and more “attracted” to the views of the extreme sides/wings of the political system (to put it very friendly).

  63. Paddy Jim Baggot MD says:

    two things almost never happen
    patients should be charged fairly-
    doctors should be paid fairly

  64. Paddy Jim Baggot MD says:

    one person told me what happened.
    the operation could have been done in office for a few hundred with local anesthesia.
    it was a five minute operation.

    instead it was done in hospital outpatient surgery.
    hospital charge more than 15,000 for 15 minute case.
    anesthesia and surgeon fees separate.

    insurance paid a third, took a third discount, and left patient with 5k bill.
    if insurance could reprice the bill for substantially less
    could the patient not do the same?

  65. Michael Gorback says:

    Dr Baggot,

    Although I decided not to comment further in this thread your comments have drawn me back in.

    One observation about your pay vs nursing pay. The nurse has to work an hour to make $40. You round on a patient who requires minimal care, spending far less than an hour and get $30. Let’s say the visit lasts 5 minutes. That’s an hourly rate of $360/hr. At 15 minutes it’s $120/hr.

    We could go into the weeds of your overhead if you’re not hospital based but I think this helps explain the disparity.

    This most recent post brings up some big questions. Although there’s often a price difference between office and hospital costs this case seems way off the charts. Of all the outpatient settings HOPD is the highest.

    There is no logical reason for this. The cost of fixing a leaking faucet at a hospital is no more that at your office, yet the cost of having identical procedures at the hospital is way higher.

    This is often justified by saying the hospital has higher overhead. To this I have two responses. (1) If the hospital can’t compete on cost then it shouldn’t be in that business. If something can be done cheaper in the office that’s where it should be done. If there are mitigating factors like poor health or extreme anxiety requiring sedation or anesthesia then the higher level of care at a facility would be justified. Not all medical offices have procedure rooms though.

    The “higher overhead” argument rings hollow for the most part. Yes, the hospital has an ER, ICU, and often has to treat indigent,, uninsured or under-insured patients, but making a patient pay higher prices to cover the cost of other services is cost-shifting. If you’re there for carpal tunnel release why are you secretly paying for the cost of an uninsured patient in the ER?

    So why do we have this huge disparity here? The hospital bills huge unrealistic prices based on the chargemaster. The insurance company contracts with the hospital and a pricing structure is negotiated. So whereas the the hospital’s “regular” chargemaster fee might be $20,000 the contract reduces it to $1,500.

    In this case the patient might have been treated at an out-of-network facility in which case the insurance might have paid much less or not at all.

    Congress passed the No Surprises Act in 2020 to address this, but SURPRISE – it doesn’t go into effect until 2022.

    In the meantime, love him or hate him, the Trump enacted a regulation that was a step toward price transparency for “shoppable” services. Hospitals must list their prices online. Compliance has been spotty. The last time I looked only 2/3 of hospitals had published their prices. Many put them online but you have to drill through their web site to find it.

    I stumbled across the disclosure portal for the Sharp Hospital system and it blew me away. You can get an estimate if you’re uninsured with a few clicks. If you have insurance you have to enter all your info and then the system calculates your estimated cost.

    I’ve played with the web sites of a few local hospitals and unlike Sharp at some point they say “call us”.

    Info on Trump’s executive order https://www.healthaffairs.org/do/10.1377/hblog20190822.353918/full/

    Info on No Surprises Act and what to do until 2022 https://pahealthaccess.org/top-ten-things-you-need-to-know-about-the-no-surprises-act/

    Sharp Hospital https://www.sharp.com/patient/billing/pricing.cfm

Comments are closed.